Sei sulla pagina 1di 88

Jeemain.

guru Exclusive Material


Downloaded from jeemain.guru

Conten
ts
PHYSI
CS

• JEE Main Solved Paper 2019 (Held on 9th January 1st Shift) ...... 3

• JEE Main Solved Paper 2019 (Held on 9th January 2nd Shift) ...... 10
• JEE Main Solved Paper 2019 (Held on 10th January 1st Shift) ...... 17

• JEE Main Solved Paper 2019 (Held on 10th January 2nd Shift) ...... 24

• JEE Main Solved Paper 2019 (Held on 11th January 1st Shift) ...... 31

• JEE Main Solved Paper 2019 (Held on 11th January 2nd Shift) ...... 39

• JEE Main Solved Paper 2019 (Held on 12th January 1st Shift) ...... 46
• JEE Main Solved Paper 2019 (Held on 12th January 2nd Shift) ...... 54
Downloaded from jeemain.guru through JEE Main 2019.
If you are willing to give your 100% , here are
the must-know-JEE Main-Preparation- Tips,
that will ease your struggle and help you work
in the right direction.

Formulate a Realistic Study


Plan

It is the most important step for the


preparation. Firstly you should be well aware
of the exam pattern and the type of questions
asked and the important topics that use to ask
in the exam. Know the weightage of each
chapter and important topics for JEE Main
exam from the graphical analysis given in this
book.

Based on your understanding, formulate a


Study Plan, by which you should be able to

N TA conducted JEE Main 2019 online


cover all important topics from each subject.

Also, it is advisable to study for all three


papers from subjects together, rather than taking one
9th to 12th January. For all the
subject at a day or for some days. Moreover,
JEE Main aspirants, now is the second now you are not left with much
chance to buckle themselves up to face fierce time to take your favourite subject first then
competition in the race to achieve their goal. come to the other subjects. Another
The second session for JEE Main 2019 is in important point is to read both class 11 and
April. There is no harm in attempting the 12 topics simultaneously. Your study plan
exam second time as this is the time to should be well-distributive. Remember to
improve your score, also if the candidate spend your Time like Money. After first
appears in both January and April sessions, session of JEE Main 2019, as you will be left
the better of the two scores will be with almost 2 months for the second session.
considered. In this short time, the best thing is that you
Now, studying and practising for JEE Main in must do the practice wisely.
a well- planned and disciplined manner can
Practice, Practice and More Practice For
only give you a good result and help you in
that, solve as much as Previous Years’
achieving success. If you practice well in this
Papers and Mock Test Papers. For online
limited time, then you can definitely make it
practice visit test.pcmbtoday.com. It is an
excellent tool to assess your preparedness tips, which we hope will help you in your JEE
with detailed report. You will get to know on Main preparation in a much easier and fruitful
which sections or topics you need to put in direction. You should keep yourself updated
more efforts and how. with the JEE news and announcements.
Difficult roads often lead to beautiful
Do Self Evaluation You are your best judge. destinations. Believe in yourself. You can do
Only you can push yourself to outperform. it!
Solve Previous Years’ Papers and Mock Test
Papers every day and see where you need All the
improvement. These are some important best!!

Must Know Tips to Score High in


JEE Main
Downloaded from jeemain.guru
9th
12Chapter Name Jan
1st Shift
th Total
Jan 2nd Shift
Question Number
Physics and Measurement – 9 6 30 23 4 – 16 6

Kinematics 19 11, 29 2, 9, 19 9, 10,


19 11 9, 21 26 14
28
Laws of Motion 23 5 – – – 7 – 1 4
Work, Energy and Power 4 27 8 13 21 – – 3 6

Rotational Motion 21, 25 14, 30 17, 18 11, 15, 6, 13,


17
10, 29 27, 30 18
3, 17, 26
Gravitation 7 20 14 20 11 – 13, 27 17 8

Properties of Solids and Liquids 15, 28 24 15, 27 1 10, 27 1, 2, 13,


14 5, 7 15
23
Thermodynamics 16 26 13 23 16 21 18 11 8
Kinetic Theory of Gases 17 22 – 2 30 – 25 9 6

Oscillations and Waves 14, 26 2, 16 10, 30 14, 19,


14, 28 5, 18,
27
29
4, 19 19
7, 16, 17
Electrostatics 20, 27,
3, 4, 13 12, 16,
29
22
8, 9, 12 8, 15,
8, 24, 26
4, 5, 12 13, 15 23
27
Current Electricity 5, 12,
23, 28 3, 23,
18
12, 21 3, 5, 18 22, 24 11, 19,
25, 28
14, 21 21
22
3, 9, 22 18, 21 1, 11 17, 29 7 10, 20
Magnetic Effects of Current and Magnetism
3, 8, 24 23, 25 18
25
2 17, 19 7 4 20, 29 12 28 6, 8 11
Electromagnetic Induction and Alternating Currents
Electromagnetic Waves 8 25 21 22 25 30 15 18 8
Optics 1, 11,
1, 15 4, 20,
30
5, 25 2, 4, 22 6, 28 1, 6 2, 24 19
24
10 12 – 18 26 19 30 10, 22 8
Dual Nature of Matter and Radiation
Atoms and Nuclei 6 6 26 7 1 16 2 29 8
Electronic Devices 24 7 29 16 24 14 23 20 8
Communication Systems – 8 5 3 15 9 26 28 7
Experimental Skills 13 10 – 6 – – 20 12 5
Total 30 30 30 30 30 30 30 30 240

Solved Paper
PHYSICS JEE 2019 MA
CHAPTERWISE ANALYSIS
9th Jan 2nd Shift
10th Jan 1st Shift
10th Jan 2nd Shift
11th Jan 1st Shift
11th Jan 2nd Shift
12th Jan 1st Shift

Downloaded from jeemain.guru

Chapterwise Graphical Analysis of JEE MAIN

JANUARY 2019
252015105
t
0
nemerusaeMd nas cisyh s
P
citameni n
K
oitoMf os wa r
L
ewoPd nay grenE, kro n
W
oitoMl anoitato n
R
oitativar s
G
diuqiLd nas diloSf os eitrepor s
P
cimanydomreh s
T
esaGf oy roehTc iteni s
K
evaWd nas noitallics s
O
citatsortcel y
E
ticirtcelEt nerru m sitengaMd nat nerruCf os tceffEc itenga s tnerruCg nitanretlAd nan oitcudnIc itengamortcel s evaWc
C M E
citp n
O
oitaidaRd nar ettaMf oe rutaNl au i elcuNd nas mot s eciveDc inortcel s noitseuQf o. o s metsySn oitacinummo s llikSl atne
D A E N C

Outcomes of Chapterwise Analysis


• Maximum number of questions were asked from Electrostatics.
• 43% of questions were asked from class XI.
• 55% of questions were asked from class XII.
• Only 2% of questions were asked from Experimental Skills.
• Weightage of Oscillations and Waves is maximum for class XI and weightage of Electrostatics is maximum for class XII.
• The six chapters with maximum weightage are Rotational Motion, Oscillations and Waves, Electrostatics, Current Electricity, Magnetic Effects of Current and Mag
(Approximately 15 questions from these chapters in each paper).

Downloaded from jeemain.guru


3 Solved Paper 2019

JEE MAIN
Held on Solved Pape
2019
9th January 1st SHIFT 1.
Consider a tank made of glass (refractive
index 1.5) with a thick bottom. It is filled with a liquid of refractive index . A student finds that, irrespective of what the incident angle i
a beam of light entering the liquid, the light reflected from the liquid glass interface is never completely polarized. For this to happen, th
of is
5 (b) 4 3 (d) 5
(a) 3 3 (c) 5 3 2. A conducting circular loop made of a thin wire, has area 3.5 × 10–3 m2 and resistance 10 . It is placed perp
time dependent magnetic field B(t) = (0.4T) sin(50 t). The field is uniform in space. Then the net charge flowing through the loop durin
10 ms is close to (a) 21 mC (b) 7 mC (c) 6 mC (d) 14 mC 3. An infinitely long current carrying wire and a small current carrying loop a
the paper as shown. The radius of the loop is a and distance of its centre from the wire is d
∝⎛⎜ a⎞⎟ ∝⎛⎜
(d > > a). If the loop applies a force F on the wire then (a) F = 0 (b) F ⎝d ⎠(c) F
a
⎝ d 2⎞3 ⎟
∝⎛⎜ a⎞⎟
⎠(d) F ⎝d ⎠2 4. Three blocks A, B and C are lying on a smooth horizontal surface, as shown in the figure. A and B have
while C has mass M. Block A is given an initial speed v towards B due to which it collides with B perfectly inelastically. The combin
5 th of the
with C, also perfectly inelastically 6 initial kinetic energy is lost in whole process.
What is value of M/m?
(a) 5 (b) 2 (c) 3 (d) 4 5. When the switch S, in the circuit shown is closed, then the value of current i will be
(a) 5 A (b) 3 A (c) 2 A (d) 4 A 6. A sample of radioactive material A, that has an activity of 10 mCi (1 Ci = 3.7 × 1010 decays/s) has tw
nuclei as another sample of a different radioactive material B which has an a
Downloaded from jeemain.guru
4
The correct choices for half-lives of A and B would then be respectively (a) 20 days and 5 days (b) 10 days and 40 days (c) 20 days and
days and 10 days
7. If the angular momentum of a planet of mass m, moving around the Sun in a circular orbit is L, about the centre of the Sun, its areal
4L (b) L (c) 2L (d) L
m m2 m m 8. A plane electromagnetic wave of frequency 50 MHz travels in free space along the positive x-directi
G ˆ G ˆ
particular point in space and time, E j = 6 3. V/m. The corresponding magnetic field B , at that point will be (a) 63 10 8 . × − k T
ˆ ˆ
k T (c) 189 10 8 . × − k T (d) 21 10 8 . × − k T 9. A current loop, having two circular arcs joined by two radial lines is shown in the f
current of 10 A. The magnetic field at point O will be close to (a) 1.0 × 10–5 T (b) 1.5 × 10–5 T (c) 1.0 × 10–7 T (d) 1.5 × 10–7 T
10. Surface of certain metal is first illuminated with light of wavelength 1 = 350 nm and then, by light of wavelength 2 = 540 nm. It is fo
maximum speed of the photo electrons in the two cases differ by a factor of 2. The work function of the metal (in eV) is close to (Energ
1240

eV )
λ ( )in nm (a) 5.6 (b) 2.5 (c) 1.4 (d) 1.8
11. A convex lens is put 10 cm from a light source and it makes a sharp image on a screen, kept 10 cm from the lens. Now a glass block
1.5) of 1.5 cm thickness is placed in contact with the light source. To get
the sharp image again, the screen is shifted by a distance d. Then d is (a) 0.55 cm towards the lens (b) 0 (c) 1.1 cm away from the lens (
from the lens
12. Drift speed of electrons, when 1.5 A of current flows in a copper wire of cross section 5 mm2, is v. If the electron density in copper
value of v in mm/s is close to (Take charge of electron to be = 1.6 × 10–19 C) (a) 3 (b) 0.2 (c) 2 (d) 0.02
13. A copper wire is stretched to make it 0.5% longer. The percentage change in its electrical resistance if its volume remains unchange
1.0% (c) 0.5% (d) 2.5% 14. A heavy ball of mass M is suspended from the ceiling of a car by a light string of mass m (m << M). When
the speed of transverse waves in the string is 60 m s–1. When the car has acceleration a, the wave- speed increases to 60.5 m s–1. The va
g (b) g (c) g (d) g
terms of gravitational acceleration g, is closest to (a) 5 20 30 10 15. Temperature difference of 120°C is maintained be
3 L
ends of a uniform rod AB of length 2L. Another bent rod PQ, of same cross-section as AB and length 2 , is connected across AB (se
steady state, temperature difference between P and Q will be close to
(a) 35°C (b) 45°C (c) 60°C (d) 75°C 16. A gas can be taken from A to B via two different processes ACB and ADB. When path ACB i
heat flows into the system and 30 J of work is done by the system. If path ADB is used work done by the system
Downloaded from jeemain.guru
5 Solved Paper 2019
is 10 J. The heat flow into the system in path ADB is (a) 100 J (b) 80 J (c) 40 J (d) 20 J
17. A mixture of 2 moles of helium gas (atomic mass = 4 u), and 1 mole of argon gas (atomic mass = 40 u) is kept at 300 K in a cont
their rm
k
(a)
θ2 (b) 3 kθ2 2
02 l kθ 2 0l(c) 0l(d) kθ 2 0l22. A bar magnet is demagnetized by inserting it inside a solenoid of length 0.2 m, 100 turns, an
current of 5.2 A. The coercivity of the bar magnet is (a) 520 A/m (b) 1200 A/m (c) 2600 A/m (d) 285 A/m
23. A block of mass 10 kg is kept on a rough inclined plane as shown in the figure. A force of 3 N is applied on the block. The coeffici
friction between the plane and the block is 0.6. What should be the minimum value of force P, such that the block does not move down
10 m s–2) (a) 25 N (b) 23 N (c) 18 N (d) 32 N
24. Mobility of electrons in a semiconductor is defined as the ratio of their drift velocity to the applied electric field. If, for an n-type se
density of electrons is 1019 m–3 and their mobility is 1.6 m2/(V-s) then the resistivity of the semiconductor (since it is an n-type semicon
contribution of holes is ignored) is close to (a) 0.2 m (b) 4 m (c) 2 m (d) 0.4 m
25. An L-shaped object, made of thin rods of uniform mass density, is suspended with a string as shown in figure. If AB = BC, and t
2 1 1 1 ⎤⎥
AB with downward vertical is , then (a) tanθ = 3 (b) tanθ = 2 (c) tanθ = 2 ⎡ is close to 3 (d) tanθ = 3 ⎢ ⎣ ⎦ (a) 2.24 (b) 0.32
18. A resistance is shown in th
and tolerance are given respectively by
(a) 27 k , 20% (b) 270 , 10% (c) 270 , 5% (d) 27 k , 10% 19. A v Gparticle = K ( yi ˆ is moving + xj ˆ ), where K with a velocity
general equation for its path is (a) y2 = x2 + constant (b) y = x2 + constant (c) y2 = x + constant (d) xy = constant 20. For a uniformly ch
R (b) R (c) R (
radius R, the electric field on its axis has the largest magnitude at a distance h from its centre. Then value of h is (a) 5 2
21. Two masses m and m/2
are connected at the two ends of a massless rigid rod of length l. The rod is suspended by a thin wire of torsional constant k at the centre
rod-mass system (see figure). Because of torsional constant k, the restoring torque is = k for angular displacement . If the rod is rotated
released, the tension in it when it passes through its mean position will be
( helium
) ( argon
)
Downloaded from jeemain.guru
626. A block of mass m, lying on a smooth horizontal surface, is attached to a spring (of negligible mass) of spring constant k. Th
spring is fixed, as shown in the figure. The block is initially at rest in its equilibrium position. If now the block is pulled with a con
maximum speed of the block is
F (b) F πF (d) 2F
(a) mk π mk (c) mk mk
27. A parallel plate capacitor is made of two square plates of side a, separated by a distance d (d << a). The lower triangular portio
dielectric constant K, as shown in the figure. Capacitance of this capacitor is
ε
K ε 0 a2 d(b) d K 1( K 0−

ln K Kε a
a 2) (c) 0

2 (d) K ε a
2 d 1( K+ ) d0 2
K
ln 28. A rod, of length L at room temperature and uniform area of cross-section A, is made of a metal having coefficient of linear ex
observed that an external compressive force F, is applied on each of its ends, prevents any change in the length of the rod, when its tem
2F F (b) (d) F
T K. Young's modulus Y, for this metal is (a) A αΔ T (c) 2 A αΔ T A αΔ
F
T A α( Δ T −273 ) 29. Three charges +Q, q, +Q are placed respectively, at distance, 0, d/2 and d from the origin, on the x-axis. I
experienced by +Q, placed at x = 0, is zero, then value of q is (a) +Q/4 (b) –Q/2 (c) +Q/2 (d) –Q/4 30. Two coherent sources prod
different intensities which interfere. After interference, the ratio of the maximum intensity to the minimum intensity is 16. The intensity
in the ratio (a) 16 : 9 (b) 5 : 3 (c) 25 : 9 (d) 4 : 1

SOLUTIONS
sin i = μ and tan r = 1 5.
1. (c) : sin r μ sin sin
1
(a) 2
2 2 2 i = 15
2 sin2 i = 2.25 2 – 2.25 sin2 i ( + 2.25)sin i = 2.25 r . tan
= 15
r
× cos r sin
. sin r sin i = 1.5 cos r
i = 1.5
cosr sini
2.
sin2 i =
25μ
2

μ 2+ 2 .25For i to be imaginary,
1−
2 sini
sin r = 1.5

1
sini 2.25 2 > 2 + 2.25
= 1.5
1.25 2 > 2

2 9
sin i .. > 5
= 1.5
3 ; minimum value of = 3
> 5 5 μμ
Δφ
2. (*) : Net charge = R 2 sin2 i2 − sin2

i = 2.25
= ( −
A B 2R B1
)

Downloaded from jeemain.guru


7 Solved Paper 2019
=
π
( 35 . × 10 − 3 )( 04 . ) ⎛⎜ ⎝sin
⎞⎟
2 − sin 0 ⎠10 = 0.14 ×

6. (a) : RA = 10 mCi, RB = 20 mCi NA = 2NB

RR
*None of the given option is correct.
3. (d) : Magnetic dipole moment of the loop, m = IA = I( a2) Force on the dipole in a non-uniform magnetic field, F = mdBdr Now, B
2 πATB
= λλ

A B NA N
B
= ( 1 /2

)
A
=(
× NN AB 12 T1 / 2

× dA =1
A 2 (T1/2)A = 4(T1/2)B 7. (b) : The area swept by the planet per unit time i.e., dt 2
r d θ 1
2 dt = 2

r 2ω ...(i)
G dA dB
Also, angular momentum L = mr2 ...(ii) Using 8. (i) and (ii), (d) : E = 6 3. Ix or, dx

=−2
μ
π0
ˆ
x I2
j dt
= 2 Lm .
m or, F a
So, F x2 d22
E = 6 3
B= c 3× 10
= 21 × 10 − 1
8 4. (d) : Ki = 2

. . 8T

mv 2 Using conservation of linear momentum


ˆ ˆ ˆ G ˆ ˆ mv
mv = (2m + M)vAs n ˆ (propagation vector) = E × B or i Hence, = B j × Bie = 21 . v1 =
(2m+
M)1
ˆ ˆ ˆ G G G G G
. ., B = k × 10 − 8 k T 9. (a) : B O= B PQ + B QR + B RS + B SP Also,
1O
( m + Mv ) 12 = 6
⎛⎜ ⎞⎟
⎝12 mv 2
⎠3 cm = 45°3 cm
+ ×
Q R or, (2m M) (2
m m + 2v
M2

)2
=

16
mv
2 2 cm 2 cm
PS
i = 10 A

or, 2
m
m +M
= 16 ⇒ Mm =
G G
4=0 440 44
5. (a) : A i1 i

⎛⎜ ⎞⎟ ˆ++ I
⎝π ⎠k μ0 π
⎛⎜ ⎞⎟ −kˆ
r2 ⎝π ⎠( ) 20 V C
= μ16
4 10 V B GO 0

⎛⎜ − ⎞⎟ k
I ⎝r 1 r2 ⎠ Let voltage at point C is VC Using KCL, i1 + i2 = i V A − 2 V C + V B − 4
2
2V=0

VC=VC−

11ˆ
π 10
BO = 4× −
7
10
× ⎡ ⎢ ⎣ 3 × 10
1

−2− 5×
10 1
−2
⎤⎥ 22
⎦= 7
× 104
⎞⎟ = ×
− 4 ⎛⎜ ⎝ ⎠ .

−5T
1240
10. (d) : E1 =
eV; E = 1240
350 2

eV 1
540 Also, v1 = 2v2 Using Einstein photoelectric equation, E – = 2

215 1 04 10 0 2 20
−2
V C + 10

−4VC= V2C
V
2(20 – VC) + (10 – VC) = 2VC VC = 10 V i = C2

−0

− −
= = mv 2 10 E 2 E
−0
5 A.

12
φ =
φ vv
22 = 4 E – = 4E – 4
12 1 2

Downloaded from jeemain.guru


8=4
E 2 − E 1 1240
3 = 3
⎛⎜
⎝540
⎞⎟ ≈ . eV = −
4− 350 1 ⎠ 1 88 11. (d) : Case I : u = –10 cm, v = 10 cm, f = ? Using lens formula, 1 f v1 u1
=
10
1 ⎛⎜ − 1 ⎞⎟ = . ⎛⎜ − 2 ⎞⎟ = 0 . 5
− − 110 f = 5 cm Case II : Due to introduction of slab, shift in the source is = t ⎝1 μ ⎠ 15 ⎝1 3 ⎠
= + =
9.5 cm, v = ? 1 v 1f u1 15

9 1.

5
=
4.
55×
d = 10.55 – 10 = 0.55 cm away from the lens.
I = 1
12. (d) : As I = neAvd = neAv v = neA 9 10 28 .
5
× × 16 . × 10 − 19 × 5 × 10 −
6

= 0.02 × 10–3 m/s = 0.02 mm/s


ρl
13. (b) : R = A
ρ ρ
2 2

For given V, dRR

= lAl
dR = 2 × (0.5) = 1.0%
= lV = 2 dll ; R
T Mg = +
14. (a) : Case I : v1 = 1μ = μ Case II : v2 = T 2 μ Ma2 g 2μ v
/
2 ) 12

⎛v⎜ ⎝v
⎞⎟ = ⎛⎜ +
⎠ ⎝2g 2 1/1

g
2
11
60 5 60 4
a
2
≈ +
⎞⎟ ⎠4 1
1
4

ag 22 . = + a2g 2 ; 1 +
a
2 g 2 a2g 2

= 30 1or
g g
a= 30 ≈ 5
15. (b) : Assume R be the thermal resistance of uniform rod of length L.A R/4 R
R R/4 R/2 P Q R/2
A B ΔT = 120
B R 3R/5 Heat current : I = R eq 8 R/5
= ⎛⎜
75 R ( T)PQ = I ⎝35
R
⎞⎟ =
⎠ 75 R
× = °C
35 R 45 16. (c) : Q = U + W; U = Q – W ( U)ACB = ( U)ADB; 60 – 30 = Q – 10
Q = 40 J
3RT
17. (d) : vrms = m
v
( rms )
He = 40 = 10 = 3 . 16 G ˆ
( vrms ) Ar 4 18. (d) : Red, violet, orange, silver R = 27 × 103 ± 10% = 27 k ± 10% 19. (a) : Here, v = K ( yi +
dy jˆ K yi ˆ xj ˆ = and dy = Kx /
dt = ( + ) dxdt Ky dt dx dy= dy dx dt
/

dt = KxKy ydy = xdx Integrating both sides ∫ ydy = ∫ xdx or y2 = x2 + constant 20. (b)
21. (c) : xCM = ml3m l 2
K
= 23 Angular frequency, = I
I = l2 =
T m/2 CM
m
×
m
m2 l ml 2
3m = 22
3
=
K K
ml 2Required tension in the rod = m = m ml
3
⎛⎜ ⎝3
2 ⎞⎟ θ l⎛⎜ ⎞⎟ = θ
⎠ 2 02 2 03 ⎝3 l ⎠ k
20
l
Downloaded from jeemain.guru
9 Solved Paper 2019
B
22. (c) : Coercivity of a bar magnet H = μ0 = nI =
N I 100
l =
× 5 . 2 = 2600 A/m
0.2
1
23. (d) : Limiting friction fs = mg cos 45° = 0.6 × 10 × 10 × 2 = 30 2 N = 42.43 N When block starts to slide downward, the downwa
block F = = = 3 + mg sin 3 + 10 × 10 73.71 N > fs × 45°
1 = 3 + 50 2
2
E
Block will not move if P = F – f P = 73.71 – 42.43 = 31.28 N 24. (d) : j = nevd Resistivity, ρ= j
= 27. (b) : x
a
dy
dx
N
P

xy
45°
d d x
y dy 1 = = a a
( dx )

dC mg cos 45°
0 0 B mg

=
Kεy
⋅ adx
+( −
εd adx y
)

1 dC = ε

⎛⎜ ⎞⎟ ∫ =
0 adx 1 ⎝K y+ d − y ⎠ dC


K yε0
adx
+ d−
a⋅

10 19 × 1 . 6 × 10 − 19 × 1 .
1 = 1
6= ne ( v d )/ E ne μ
⎛⎜
ey ⎝K
1 ⎞⎟
−1 ⎠= 0.39 m 0.4 m
ε 21
= 0 1 0 25. (d) : Assume m be the mass of one rod. At equilibrium,
+ =0 L
(rod 1) (rod 2) or mg
L
mg L
d
⎛⎜ ⎞⎟
⎝−
⎡⎢ ln ⎛⎜ ⎛⎜ ⎞⎟ ⎞⎟ ⎤ ⎥
⎠ ⎣ ⎝+ ⎝− ⎠ ⎠ ⎦ =−
d
a11K

dyK
⎛⎜ ⎛⎜ ⎞⎟
⎜ ⎝+ ⎝− ⎠⎞( 1 K ε0 K a) 2 d
ln

d d 1 Kd

1 ⎟⎟ ⎠⎛⎜ ⎝2
sin
θ
⎞⎟
⎠( 1
K−
ln
ε 0 K a) 2 d ⎛⎜ ⎝K 1
⎞⎟ =
⎠ K(ε K0a −2
ln
1 )d
K
⎛⎜
− ⎝2 cos θ − sin
θ
⎞⎟ =

0

3
or 2 2

28. (b) : As the temperature of rod rises, Extension in the rod, L = L T L s


F L
, tan θ = 13 Young’s modulus Y = A
L
F A
of the rod, Y = / ΔL/L
F ⎛⎜ 1
26. (a) : Extension in spring, x = Using work energy theorem k WF + Wspring = KE Fx + ⎝− 2
⎞⎟ = −
kx ⎠ mv × Δ = AL L Δα ×
F
QQ
T 29. (d) : d
2+ =02 ⎛⎜
( d Qq )/ 2 2 12 2 0 F ⎝Q + 4q
F ⎞⎟ − 1
k ⎠ 2
k ⎛⎜ F I
⎝ k 30. (c) : max
⎞⎟ = mv = 16 or A A
Imin ⎠ 1 ⎛ ⎜ ⎝A +


A 22
⎞⎟
⎠1 1 2
2
=
2

161
F
v = mk

12
A1 + A2 = 4A1 – 4A2 3A1 = 5A2 or A1A2
I
= 53 1I2
=
259

Downloaded from jeemain.guru


10 JEE Main

JEE MAIN
Held on Solved Pape
2019
9th January 2nd SHIFT 1.Two plane mirrors are inclined to each other
such that a ray of light incident on the first mirror M1 and parallel to the second mirror M2 is finally reflected from the second mirror M
first mirror (M1). The angle between the two mirrors will be (a) 75° (b) 90° (c) 60° (d) 45° 2. A particle is executing simple harmonic m

amplitude A, along the x-axis, about x = 0. When its potential energy (PE) equals kinetic energy (KE), the position of the particle will b
(c) A (d) A
2 2 2 3. A parallel plate capacitor with square plates is filled with four dielectrics of dielectric constants K1, K2, K3 and K4 a
= K + KK+
in figure. The effective dielectric constant K will be (a) K K + K
K ⎡ ⎢ 1
+ K + K (in V/m) at a point y = 3 m on y-axis is take 4 ⎣ πε
0
= 9 × 10

9N m2C−
⎤ ⎥
2 ⎦ (a) ( 63 i ˆ − 27 j ˆ ) × 102 (b) ( 81 i ˆ − 81 j ˆ ) × 102 (c) ( − 63 i ˆ + 27 j ˆ ) × 102 (d) ( − 81 i ˆ + 81 j ˆ ) × 102 5. A mass of 1
suspended vertically by a rope from the roof. When a horizontal force is applied on the rope at some point, the rope deviated at an angle
roof point. If the suspended mass is at equilibrium, the magnitude of the force applied is (g = 10 m s–2) (a) 140 N (b) 70 N (c) 100 N (d
RB
given instant, say t = 0, two radioactive substances A and B have equal activities. The ratio RA
of their activities after time t itself
decays with time t as e–3t. If the half life of A is ln2, the half-life
)1234
=2
(b) K
K ln2 (b) ln2 (c) 4 ln2 (d) 2 ln2
(( K 1 1 (a) 4 2 7. Ge and Si diodes start conducting at 0.3 V and 0.7 V respectively. In the follo
diode connection are reversed, the value of V0 changes by (assume that the Ge diode has large breakdown voltage)
K
4
)K 2+K3+
= K + K
K 4 ) (c) K (K 11+ K 23

K + K K +
(( K 11 + 4 )( 2

K
3
)K 2+K3+
K 4)
4. Two point charges q1 10( μC ) and q2(–25 C) are placed on the x-axis at x = 1 m and x = 4 m respectively.
Downloaded from jeemain.guru
11 Solved Paper 2019
(a) 0.4 V (b) 0.2 V (c) 0.6 V (d) 0.8 V 8. In a communication system operating at wavelength 800 nm, only one percent of source frequ
as signal bandwidth. The number of channels accomodated for transmitting TV signals of bandwidth 6 MHz are (Take velocity of light
h = 6.6 × 10–34 J s) (a) 3.75 × 106 (b) 4.87 × 105 (c) 6.25 × 105 (d) 3.86 × 106 9. Expression for time in terms of G (universal gravitation
hc5 Gh Gh (d) c3
(Planck’s constant) and c (speed of light) is proportional to (a) G(b) c3 (c) c5 Gh10. The pitch and the number of d
circular scale, for a given screw gauge are 0.5 mm and 100 respectively. When the screw gauge is fully tightened without any object, th
circular scale lies 3 divisions below the mean line. The readings of the main scale and the circular scale, for a thin sheet, are 5.5 mm an
the thickness of this sheet is (a) 5.950 mm (b) 5.755 mm (c) 5.725 mm (d) 5.740 mm 11. The position co-ordinates of a particle moving
coordinate system is given by x = a cos t, y = a sin t and z = a t. The speed of the particle is (a) 2a (b) 3 aω (c) 2 aω (d) a 12. The m
associated with a light
wave is given, at the origin, by B If this = B0light [sin(3.14 falls on × 10a silver 7)ct + plate sin(6.28 having
function of 4.7 eV, what will be the maximum kinetic energy of the photo electrons? (c = 3 × 108 m s–1, h = 6.6 × 10–34 J s) (a) 12.5 eV
6.82 eV (d) 7.72 eV
13. Charge is distributed within a sphere of radius R with a volume charge density
A
ρ( r ) = r 2
e−
r/a
2 , where A and a are

constants. If Q is the total charge of this charge distribution, the radius R is (a) a log ⎛⎜⎜

⎝1 −
1
2
log ⎛⎜ 1 − Q ⎞⎟ a log ⎛⎜ 1 − Q
QaA ⎞π ⎟⎟ ⎠(b) a ⎝ 2π aA ⎠(c) 2 ⎝ 2π
log
⎛⎜⎜ ⎝1 −

2
1

π QaA ⎞2 ⎟⎟ ⎠14. A rod of length 50 cm is pivoted at one end. It is raised such that it makes an angle of 30° from the horizontal a
–1 –2 30 (b) 30
released from rest. Its angular speed when it passes through the horizontal (in rad s ) will be ( g = 10 m s ) (a) 2 2 (c) 30 (d)
Young’s double slit experiment, the slits are placed 0.320 mm apart. Light of wavelength = 500 nm is incident on the slits. The total nu
fringes that are observed in the angular range –30° 30° is (a) 641 (b) 321 (c) 640 (d) 320 16. A musician using an open flute of length 5
second harmonic sound waves. A person runs towards the musician from another end of a hall at a speed of 10 km/h. If the wave speed
frequency heard by the running person shall be close to (a) 500 Hz (b) 753 Hz (c) 666 Hz (d) 333 Hz 17. A power transmission line fee
2300 V to a step-down transformer with its primary windings having 4000 turns. The
Downloaded from jeemain.guru
12
output power is delivered at 230 V by the transformer. If the current in the primary of the transformer is 5 A and its efficiency is 90%, t
would be (a) 25 A (b) 50 A (c) 45 A (d) 35 A 18. One of the two identical conducting wires of length L is bent in the form of a circular
other one into a circular coil of N identical turns. If the same current is passed in both, the ratio of the magnetic field at the centre of the
B will be 1 1
at the centre of the coil (BC), i.e. BLC (a) 2N (b) N2 (c) N (d) N 19. A series AC circuit containing an inductor (20 mH), a
F) and a resistor (60 ) is driven by an AC source of 24 V/50 Hz. The energy dissipated in the circuit in 60 s is (a) 3.39 × 103 J (b) 5.65 ×
103 J (d) 5.17 × 102 J 20. The energy required to take a satellite to a height h above Earth surface (radius of earth = 6.4 × 10 3 km) is E1
energy required for the satellite to be in a circular orbit at this height is E2. The value of h for which E1 and E2 are equal is (a) 3.2 × 103
104 km (c) 1.6 × 103 km (d) 6.4 × 103 km 21. A particle having the same charge as of electron moves in a circular path of radius 0.5 cm
influence of a magnetic field of 0.5 T. If an electric field of 100 V/m makes it to move in a straight path, then the mass of the particle is
electron = 1.6 × 10–19 C) (a) 2.0 × 10–24 kg (b) 1.6 × 10–19 kg (c) 1.6 × 10–27 kg (d) 9.1 × 10–31 kg 22. A 15 g mass of nitrogen gas is en
at a temperature 27°C. Amount of heat transferred to the gas, so that rms velocity of molecules is doubled, is about [Take R = 8.3 J/K m
(b) 10 kJ (c) 0.9 kJ (d) 6 kJ 23. A carbon resistance has a following colour code. What is the value of the resistance ?
(a) 64 k ± 10% (b) 530 k ± 5% (c) 5.3 M ± 5% (d) 6.4 M ± 5% 24. The top of a water tank is open to air and its water level is maintaine
out 0.74 m3 water per minute through a circular opening of 2 cm radius in its wall. The depth of the centre of the opening from the level
tank is close to (a) 9.6 m (b) 6.0 m (c) 2.9 m (d) 4.8 m 25. The energy associated with electric field is (UE) and with magnetic field is (
U
electromagnetic wave in free space. Then (a) UE > UB (b) U
B
E = 2 (c) UE = UB (d) UE < UB 26. Two Carnot engines A and B are operated in series. The first one, A, receives heat at T1 (= 600 K

reservoir at temperature T2. The second engine B receives heat rejected by the first engine and, in turn, rejects to a heat reservoir at T3
Calculate the temperature T2 if the work outputs of the two engines are equal. (a) 500 K (b) 300 K (c) 400 K (d) 600 K 27. A force acts
so that its position is given as a function of time as x = 3t2 + 5. What is the work done by this force in first 5 seconds? (a) 850 J (b) 950
900 J 28. In the given circuit the internal resistance of the 18 V cell is negligible. If R1 = 400 , R3 = 100 and R4 = 500 and the reading o
voltmeter across R4 is 5 V, then the value of R2 will be
(a) 450 (b) 550 (c) 230 (d) 300
Downloaded from jeemain.guru
13 Solved Paper 2019
29. In a car race on straight road, car A takes a time t less than car B at the finish and passes finishing point with a speed v more than th
a 1+ a 2
the cars start from rest and travel with constant acceleration a1 and a2 respectively. Then v is equal to (a) 2 t (c) 2 aa 1 2 t
1212
30. A rod of mass M and length 2L is suspended at its middle by a wire. It exhibits torsional oscillations. If two masses each of m are a
distance L/2 from its centre on both sides, it reduces the oscillation frequency by 20%. The value of ratio m/M is close to (a) 0.17 (b) 0.

0.57 SOLUTIONS
1. (c) : M1
M2 + + = 180° 3 = 180°; = 60° 2. (b) : Given, Potential energy = Kinetic energy
12
aaa+
t =
a E 1 4 πε 1
0
q 10 × 10
1r 12 = 9 × 10 9 × 10

6

=
900 10
V/ m
= × 1
E2 cos θ9= 10 10
× 25 × 10
9, sin θ 25 = −

310 6
=
9000
V/ m

, cos α = 45

, sin α = 35

m or, 2 x 2 ω 2 x 2 ω= = A 2 ;

( A 2− x 2)
G G G ˆ ˆ ˆ ˆ ˆ
Net electric field, E =E 1+ E 2= E 1 x ( − i ) + E 1 y j + E 2 x i + E 2 y ( − j ) = ( − E 1 cos θ + E 2 cos α ) i + ( E 1 sin

3. (*) : C 1
2)
/2

ˆ ˆ ˆ
C1C2= ( − 900 + 7200 ) i + ( 2700 − 5400 ) j = 6300 i + ( − 27
ε
C 1=
0K A
d1

= ( 63 i ˆ − 27 j ˆ ) × 102 V/m C3
C4
ε 0K A
5. (c) : At equilibrium C 2 = d2
C ε 0K A
, 3= d3
,
C ε 0K A CC
4= d4 CC
C
mg
T F cos = T θ = mg sin θ = ; cos T =
mg
θ cos × sin θ θ
T cos
10 ×
= cos
10
45
× sin
°
45 °
mg

= 100 N
RA
6. (a) : At t = 0, R0B = R0A At time t, RB
e = 12
3T eq

1+ 2
+
C C 33C
4+ C
4
=
⎛⎜ ⎝K K 1 1 K
2+ K
2
+
K K 33+ K
K4
4
⎞⎟ ε

0d AT sin F As Ceq = ε0K eq A d*None of the given options Keq = is K correct.


+
K 11 K
K2
+
23 K K 33K
4+

K 4 4. (a) : q1 = y

10 μC, q2 = –25 C
E1
E1y
q
1
q Ae
21 m 2 m 3 m 4 mx = 3 t E1xE2x R
E2
e −
R B e t 0t 0 λA− λ B

=
3 t E2y

e −(λ A−λ B)t=e

t
⇒λ A− λ
B

=−
Downloaded from jeemain.guru
14
T
ln 2 ( 12 )

−T =− T =
A 12 2 B 3 12 2 B

4
T ln
[ (T1/2)B = ln 2] ( 1/2) B

; ln = 2
ln / ( / ) ( /) 4 7. (a) : Case I : Initially diode Ge is conducting Case so, II : VAs 0 = Ge 12 diode – VGe is = reversed 12 – 0.3 bi

Si diode starts conducting. Hence, V0 = = 12 12 – – V0.7 Si = 11.3 V Required change in V0 = 11.7 – 11.3 = 0.4 V 8. (c) : = 800 nm,
c
6 MHz = 6 × 106 Hz Operating frequency, υ = λ =8
3 1015
9= 8× Hz Number of chan
1
=
%
υΔ
of υ

=
38

×1010 1526 × 10 6 = × 7 = . × 5 9. (c) : For time, t = cx hy Gz [M0L0T1] = [LT –1]x [ML2T –1]y [M–1L3T–2]z Using principle of hom
dimensions
y – z = 0 ...(i) x + 2y + 3z = 0 ...(ii) –x –y –2z = 1 ...(iii) On solving eqn
x =− 2
5 = Gh Pitch 0.
y = z = 116 10 6 25 10 , 12 Hence, t c 5 10. (c) : Pitch = 0.5 mm, N = 100 L.C. = Thickness N of sheet =
5 mm 5 10
100 = MSR = + CSR × × − 3 LC mm ± Zero Error = (5.5 + 48 × 5 × 10 –3 – 3 × 5 × 10–3) mm = (5.5 + 0.225) mm = 5.725 m
= : a a x = sin t; a vz cos t; = va
y= t; a vx = cos –a sin t
t
Speed of particle, v = v 2 x + v 2 y + v z 2
=a 2ω 2+a 2ω 2=a ω
2
12. (d) : = 4.7 eV Frequency of light used for maximum energy
×
υ = ( 6 . 28 10
7
)
c2×
×
314. = 107 × 3 × 108 = 3 × 1015 Hz E = h = 6.6 × 10–34 × 3 × 1015 J = × 66 .
×
3
10

16 . 10

19
19
eV = 12.375 eV

Using Einstein’s photoelectric equation,


Kmax = E – = 12.375 – 4.7
= 7.675 eV 7.72 eV
A
13. (d) : ρ( r ) = r 2

e−

⎛⎜
= ρ( r ) dV = ∫
Ae − r a
2
r/aQ
∫ ⎝r
⎞⎟ × r dr R
⎠ Or
R
dr
2
/
2 4π
2
0

R=
(
4πA
)
∫ e−2
r / a dr 0 =

a ⎞⎟ ⎡
⎠ ⎣ e−2
r/

a⎤ ⎦ 0
RQ = − 2 π Aa [ e − 2 R / a − e − 0 ] − 2 π
Q e−2
Aa =
R/

a− 1
e −2R
Q
π Aa

log
11−
1
Q /( 2 π Aa ) 14. (c) : l = 50 cm, h = 25 sin 30° = 12.5 cm Potential energy at top = Kinetic energy at botto or, mgh = 2
⎞ ⎟ 1 1
I ⎛⎜ ⎝ 2 ⎠ω 2mgh = 2 × ml gh = 12 6

3 l 2 ω 2ω;
= ω 1 l 26 gh CM
h
× × ×
ω=6 10 125 .
10

2
50 × 10 −
2
=

30

rad/s
dsinθ
15. (a) : d = 0.320 mm, = 500 nm Path difference, d sin = n Maximum value of integer, n =
λ
Downloaded from jeemain.guru
15 Solved Paper 2019
0 32 10
= . × −
3
×
1
( /
2
500 × 10

9
=
) 1600
=
5
320

Hence, total number of maxima observed in


GMm
20. (a) : E1 = Ef – Ei E R h angular range –30° 30° is = 32
16. (c) v = : 330 l = 50 m/s
25 m/s
cm = 0.5 m, vp =10 km/ h = 9 Frequency of second harmonic produced by an open flute,

υ= 2 × 2
= − ⎞⎟ +
==1 ⎛⎜ ⎝ ( + ) ⎠ GMmR

2( R + h ) As per question, E1 = E2 − ( R GMm + h


+
) GMmR
=
2

GMm
660 Hz
0.5 Frequency heard by the person

1R=2(R3
h R 32 103
+ h ) or , = 2= .×
km
= mv
21. (a) : r
...(i)
qB
v+v
υ′= 0v

υ=
25
330 9
660 666 Hz
330
rqB ×B=E
Also, FB = FE or, qvB = qE [Using (i)] or m 17. (c) : p = 23
= rqB =P
m s= 230 V, Ip = 5 A E = 90% = 0.9, Is = ? η P
= ε
ε +
×≈
2
s Is
× × × ×
m= 05 . 10 − 2 16 . 10 − 19 ( 05

p
××
= =×
Is
= ηε I
oi p

εsp
= 0 . 9 × 2300 × 5

45
230 = A.
08 . 025 . 10
2 10
− 21 100

− 24 kg
22. (b) : m = 15 g, T = 27°C = 300 K
B
18. (a) : LB


C=
R0
⎛⎜
L I×

⎝N 2 Rμ

C0I

= ⎛⎜
⎞⎟ ⎠
×
⎝RCR L ⎞⎟ ⎠ N

1 ...(i)
;
T′=4
15 R 15
n = 14 Q = U = n CV T As L = 2 RL = N(2 RC) or CR L = 28

1 ⎛⎜ ⎞⎟ ( −)
=N ⎝ ⎠ From eqn. (i) and (ii), B...(ii)

15
52 R 4 1 300 = 28 ×
1
2

52 830 . 3 300 R = 10004 J 10 kJ 23. (b) : G O Y Golden 19. (d) : Impedance, Z = R 2 + ( X C − X L ) 2 XL = L = (2 L) = 6.28 × 50 ×

1
πυ

C = 628 . × 120 1
26 54
× 10 − 6 × 50 = . Ω
5 3 104 5% R = 53 × 104 + 5% = 530 k + 5%
dV = A dx = A 2 gh
24. (d) : As, dt dt Z = ( 60 ) 2 + ( 20 . 26 ) 2 ; Z2 = 4010 2 Average power dissipated, Pav = rms Irms cos

hA
ε
= ε rms
rms
Z
12
=
⎛⎜ ⎞⎟ × 1
⎝dV2 dt ⎠ 2 g 2=
⎛⎜
⎝227 1
× 4 × 10

⎞⎟
4
⎛⎜ ( 24
⎠2 × ⎝=
) ε
2 4010 × Z R= 2
rms Z 2R×

60 8 62
× W= .
W
Energy dissipated in 60 s = 8.62 × 60 = 5.17 × 102 J
0 60 .
74
⎞⎟ ×
⎠ 1
2 × 98 .
××
=
× ⎛⎜ ⎞⎟ × ≈
×× ⎝ ⎠ × 49 10 8 10
− 4 22 22 16
742
1 60
4.9
2 98 .
m

Downloaded from jeemain.guru


16 25. (c)
W
26. (a) : For Carnot engine A, Q
⎛⎜
=
W
⎝1 − 21 ⎞⎟ ⎠For Carnot engine B, Q

AT1 T

⎛⎜
⎞⎟ ⎠As per equation, WA = WB Q 1
1−T
⎝ T 21 B2
Q 2 1 T3 ⎛⎜
T 2=

⎝1 − T T32 ⎞⎟ ⎠TT ⎞⎟ ⎠
'
⎟ ⎠TT
⎛⎜
⎝1 − TT 21
= ⎛⎜
⎟⎠
− ⎛⎜
⎝1 TT 2 3⎞⎟ ⎠

⎝Q1Q 2
=
T T12 1T2
−1=1−
= T 1+
T 2 3T 2 2
T
3

+
= 600 2 400
500
= K
27. (d) : x = 3t2 + 5; m = 2 kg
= dx =6t 1 2 6 5 1
v dt Work done = change = 2 × [( × ) 2 in KE = 2
6 0 2 900
m (v 2f− ( × ) ]= J

− v i2)
28. (d) : R1 = 400 I1
R5
R1 I2I
18 V
R2
R5 = R3 + R4 = 100 + 500 = 600 Voltage across R4, V4 = I1R4
or I1 = 500 5Voltage across = 100 1A

R3, V3 = I1R3 = 1100 × 100 = 1 V V2 = V3 + V4 = 6 V or I2R2 = 6 ...(i)


2 A
V1 = 18 – 6 or IR1 = 12 or I2 = I – I1 = 100
12
I=
400
= 100 3A
6I =
From eqn. (i) R2 = 2 300
Ω

29. (b) : Both cars cover same distance.


12

a 1 t 12 = 12 at 2 22 ...(i) Also, t2 – t1 = t ...(ii) a1t1 = v + a2t2 ...(iii) From


⎛ ⎜
a ⎝a
= +⎛⎜
⎞⎟ ⎠
⎝t t t
⎞⎟ ⎠2
= ⎛⎜

⎝1
+
1 ⎞⎟ ⎠2

tt
11 t2 1

t
1

aa t a 1+ = 1;
a
1= 2

t
a 2 1 − 2 From eqn. (ii) and (iii), a1t1 = v + a2 (t + t1); v +
+ = −
or v a 2t (a1 a 2)
×
( a2
t
a 1−

a2 ) v + at 2 = aa 1 2 t + at 2
v = aa 1 2 t 30. (c) : For torsional pendulum, oscillation frequency
υ=2
1 1
π CI For given C, υ∝ I υ1υ2
I
= 2I 1 ...(i)
= 1 (2 ) =1 ,I ML 2 m ⎛⎜ L
I1 12 M L 2 3 ML 2 2= 13 +2 ⎝
⎞⎟ 1 mL
2 ⎠2 = 3 ML 2 +
2
2

= % of =4
υ2 80 υ1 5 υ 1 Using these values in eqn. (i),

54
=

mL 2 ⎞⎟
13 ML 2 + 2 13ML 2 ⎛⎜ ⎝54
3 m⎞⎟ or, 25= 1 + 3 m m
M ⎠ 16 2 M 2 3M = 16
9or, mM = 38 = 0 .
37
Downloaded from jeemain.guru
17 Solved Paper 2019

JEE MAIN
Held on Solved Pape
2019
10th January 1st SHIFT 1. magnet of total magnetic moment
ˆ
10 − 2 i ˆAm 2 is placed in a time varying magnetic field, Bi (cos ω t ) where B = 1 Tesla and = 0.125 rad/s. The work done for rever
of the magnetic moment at t = 1 second, is (a) 0.007 J (b) 0.01 J (c) 0.028 J (d) 0.014 J 2. A piece of wood of mass 0.03 kg is dropped f
100 m height building. At the same time, a bullet of mass 0.02 kg is fired vertically upward, with a velocity 100 m s –1, from the ground
embedded in the wood. Then the maximum height to which the combined system reaches above the top of the building before falling b
s–2) (a) 10 m (b) 30 m (c) 20 m (d) 40 m 3. A uniform metallic wire has a resistance of 18 and is bent into an equilateral triangle. Then,
between any two vertices of the triangle is (a) 4 (b) 2 (c) 8 (d) 10
4. A plano convex lens of refractive index 1 and focal length f1 is kept in contact with another plano concave lens of refractive index 2 a
f2. If the radius of curvature of their spherical faces is R each and f1 = 2f2, then 1 and 2 are related as (a) 3 2 – 2 1 = 1 (b) 1 + 2= 3 (c) 2 1
= 1 5. A TV transmission tower has a height of 140 m and the height of the receiving antenna is 40 m. What is the maximum distance u
signals can be broadcasted from this
tower in LOS (Line of Sight) mode? (Given : radius of earth = 6.4 × 10 6 m) (a) 65 km (b) 48 km (c) 40 km (d) 80 km 6. The density of
units is 128 kg m–3. In certain units in which the unit of length is 25 cm and the unit of mass is 50 g, the numerical value of density of t

640 (b) 16 (c) 40 (d) 410 7. A solid metal cube of edge length 2 cm is moving in a positive y-direction at a constant speed of 6 m/s . T
magnetic field of 0.1 T in the positive z-direction. The potential difference between the two faces of the cube perpendicular to the x-axi
g
6 mV (c) 1 mV (d) 12 mV 8. A starts block from of mass rest with m is kept constant on a acceleration platform which 2 upwards, as

figure. Work done by normal reaction on block in time t is


mg 2 t2
(a)
(b) 0 3
8 (c)

mg 2 t 2 8(d) − mg 8
2 2
t
9. In the cube of side a shown in the figure, the vector from the central point of the face ABOD to the central point of the face BEFO w
Downloaded from jeemain.guru
18
1 a(jˆ kˆ) 1 a(ˆj iˆ) 1 a(kˆ iˆ) 1 a(iˆ kˆ)
(a) 2 − (b) 2 − (c) 2 − (d) 2 − 10. A train moves towards a stationary observer with s
train sounds a whistle and its frequency registered by the observer is f1. If the speed of the train is reduced to 17 m/s, the frequency regi
f1 is 20(b) 18 (c) 21(d) 19
speed of sound is 340 m/s, then the ratio f2 (a) 19 17 20 18
= x on it. The rod is
11. An insulating thin rod of length l has a linear charge density ρ ( )x ρ0 l rotated about an axis passing through

and perpendicular to the rod. If the rod makes n rotations per second, then the time averaged magnetic moment of the rod is (a) n l3 (b)
π
ρ n l 3 (c) 4

n ρ l 3 (d) n l3
12. A parallel plate capacitor is of area 6 cm2 and a separation 3 mm. The gap is filled with three dielectric materials of equal thickness
dielectric constants K1 = 10, K2 = 12 and K3 = 14. The dielectric constant of a material which when fully inserted in the given capa
capacitance would be
(a) 36 (b) 12 (c) 4 (d) 14 13. Three Carnot engines operate in series between a heat source at a temperature T1 and a heat sink at tem
figure). There are two other reservoirs at temperature T2 and T3, as shown, with T1 > T2 > T3 > T4. The three engines are equally

(a) T 2 = ( T 12 T 4 ) 1 / 3 ; T 3 = ( TT 1 4213 ) / (b) T 2 = ( TT 1 4 ) 1 / 2 ; T 3 = ( T 12 T 4 ) 1 / 3 (c) T 2 = ( T 13 T 4 ) 1 / 4 ; T 3 = (

T 2 = ( TT 1 4213 ) / ; T 3 = ( T 12 T 4 ) 1 / 3 14. A satellite is moving with a constant speed v in circular orbit around the earth. An ob

ejected from the satellite such that it just escapes from the gravitational pull of the earth. At the time of ejection, the kinetic energy of th

15. A mv 2 heat source (b) 2 mv2 (c) at T = 103 32

mv 2 (d) K is connected
mv2
to another heat reservoir at T = 102 K by a copper slab which is 1 m thick. Given that the thermal conductivity of copper is 0.1 W K –1

flux through it in the steady state is (a) 120 W m–2 (b) 90 W m–2 (c) 200 W m–2 (d) 65 W m–2 16. Two dipole electric moments dipoles
ˆ G ˆ
qai and with d B respective = −2 qai are placed on the x-axis with a separation R, as shown in the figure

Downloaded from jeemain.guru


19 Solved Paper 2019
The distance from A at which both of them
R (b) R 2R (d) 2R
produce the same potential is (a) 2 1− 2 1+ (c) 2+1 2 − 117. To mop-clean a floor, a cleaning machine presses a
radius R vertically down with a total force F and rotates it with a constant angular speed about its axis. If the force F is distributed un
μFR (b) μ
mop and if coefficient of friction between the mop and the floor is , the torque, applied by the machine on the mop is (a) 2
(d) 2 μFR
3 18. A homogeneous solid cylindrical roller of radius R and mass M is pulled on a cricket pitch by a horizontal force. A
F (b) F 2
without slipping, angular acceleration of the cylinder is (a) 2 MR 3 MR (c) 3 then the maximum electric field associated with
N/C (b) 4 × 104 N/C (c) 6 × 104 N/C (d) 3 × 104 N/C
22. A charge Q is distributed over three con- centric spherical shells of radii a, b, c (a < b < c) such that their surface charge densities
Q Qa
another. The total potential at a point at distance r from their common centre, where r < a, would be (a) 4 πε 0 ( a + b + c ) (b)
ca
12 πε 0 abc

) 4 πε

0(a 2+ b 2+ c
Q a b
2 ) (d) ( 2+ 2+

c
2
)4
πε
0( a 3+b 3+c
3
)
23. A potentiometer wire AB having length L and

resistance 12r is joined to a cell D of emf and internal resistance r. A cell C having emf 2 εand internal resistance 3r is connected.
3
FMR (d) F2

MR
The length AJ at which the galvanometer as shown in figure shows no deflection is
19. Two guns A and B can fire bullets at speeds 1 km/s and 2 km/s respectively. From a point on a horizontal ground, they are fired in
directions. The ratio of maximum areas covered by the bullets fired by the two guns, on the ground is (a) 1 : 2 (b) 1 : 16 (c) 1 : 4 (d) 1 :
electron microscope, the resolution that can be achieved is of the order of the
11L (b) 5L
(a) 12 12 wavelength of electrons used. To resolve a width of 7.5 × 10 –12 m, the minimum electron energy required is clos

(b) 100 keV (c) 1 keV (d) 500 keV 21. If the magnetic field of a plane electromagnetic wave is given by (The speed of light = 3 × 10 8 m
⎡ ⎢
10 − 6 sin ⎣ 2π
× × ⎛⎜ −x 11 L (d) 13 L
2 10 15 ⎝t c (c) 24 24
1 rad by using ligh
24. In a Young’s double slit experiment with slit separation 0.1 mm, one observes a bright fringe at angle 40
is used a bright fringe is seen at the same
When the light of wavelength 2 angle in the same set up. Given that 1 an

Downloaded from jeemain.guru


20
are in visible range (380 nm to 740 nm), their
28. In the given circuit the cells have zero internal values are
resistance. The currents (in amperes) passing (a) 400 nm, 500 nm (b) 625 nm, 500 nm
through resistance R1 and R2 respectively, are (c) 380 nm, 525 nm (d) 380 nm, 500 nm
25. A 2 W carbon resistor is colour coded with green, black, red and brown respectively. The maximum current which can be passed th
resistor is (a) 20 mA (b) 0.4 mA (c) 100 mA (d) 63 mA
26. Using a nuclear counter the count rate of emitted particles from a radioactive source
(a) 0, 1 (b) 1, 2 (c) 2, 2 (d) 0.5, 0 29. To get output 1 at R, for the given logic gate
circuit the input values must be
is measured. At t = 0 it was 1600 counts per Psecond and t = 8 seconds it was 100 counts per second. The count rate observed, as
Q
counts per second, at t = 6 seconds is close to (a) 200 (b) 360 (c) 150 (d) 400
27. Water flows into a large tank with flat bottom at the rate of 10 –4 m3 s–1. Water is also leaking out of a hole of area 1 cm2 at its botto
of the water in the tank remains steady, then this height is (a) 4 cm (b) 5.1 cm (c) 2.9 cm (d) 1.7 cm

SOLUTIONS
R
(a) X = 1, Y = 0 (b) X = 1, Y = 1 (c) X = 0, Y = 1 (d) X = 0, Y = 0 30. A string of length 1 m and mass 5 g is fixed at both ends. The ten
is 8.0 N. The string is set into vibration using an external vibrator of frequency 100 Hz. The separation between successive nodes on the
to (a) 10.0 cm (b) 16.6 cm (c) 20.0 cm (d) 33.3 cm
1. (d) : Required work done, W = 2 m B cos t = 2 × 10–2 × cos[(0.125) × 1]
0.02 J 2. (d) : Suppose both collides at point P after time t. Time taken for the particles to collide,
= d
t v rel

==

before and after the collision.


– (0.03) (10) + (0.02) (90) = (0.05)v 150 = 5v v = 30 m/s
(100 – h)
v 1s
Maximum height reached by body H = 22 g100 100
h
P
30 × 30
H= 2 × 10
= 45 m

1
(100 – h) = 2 Speed of wood just before collision = gt = 10 m/s Speed of bullet just before collision v – gt = 100 – 10 = 90 m/s
Before After 0.03 kg 10 m/s v 0.02 kg 90 m/s 0.05 kg Now, conservation of linea
h = 95 m

gt 2 = 12 × 10 × 1
Bullet
Height above tower = 40 m
ρl
3. (a) : For given wire, 18 Ω = A For the two vertices A and
B
...(i)

C, AB and BC are in series and this combination is in parallel with AC. A C


Downloaded from jeemain.guru
21 Solved Paper 2019
Net resistance =
ρ lρ lρ

A ρ lρ ρ
lA
a a a a
9. (b) : The position vectors for points G and H are 2k 2i 2j 2kA
/3/3/
3
/3/3/
3
=
⎞⎟ ⎛⎜ + ⎞⎟ ⎛⎜
⎛⎜ ⎝ ⎠ ⎝ ⎠+ ⎝
⎞⎟ and ⎛⎜ + ⎞⎟ respectively.
⎛⎜ ⎝ + ⎠ ⎝ ⎠ So, the displacement vector from
⎞⎟ ⎛⎜
l/ 3 ⎠ ⎝ρ

lA /
⎞⎟
3 ⎠3 ρ
18 ⎛⎜ 2
l/ 3 = ⎝ 9

A
⎞⎟ [Using (i)]

a a
OH − OG = 2( k + j − k − i ) = 2( j − i ) 10. (d) : Let f1 and f2 be the frequency registered = 4 4. (c) : As per lens maker’s form
− ) ⎛⎜ R
1 ⎝ 1 1 by the observer in two cases.
v +v
f 1= f
0 = f 340 +0 = f−R ⎞
v − vS 340 − 34 − 12
RR(μ11
) ( μ 2 ) 340 340 34
1 (f 2 (f2)
1)

= −
f2 f 340 0
=
340 − 17 − 2

f
1 The required ratio 1f2
N

mg mg/2 2 2 2 = 340 340 − 17 34
=
1918 ( 2 – 1) = 2( 1 – 1) 2 1 – 2 = 1
11. (c) : Consider an element dx at distance x from origin.
dx 5. (a) : The maximum distance signals can be broadcast dm is
upto which
O

d m = Rh T +
Rh R = × × +
x
22
2 64 . 10 6 ( 140 40 ) m 65 km 6. (c) : For the two system of units,
The current produced due to this moving element is
= = ρ x dx
di ndq n 0 l N1u1 = N2u2 N2
128 kg m
3= N2
( 25 50
cm g
)3
;

The magnetic moment of the rod is



∫ ( πx 2 ) di = n ρ0 l π
x . x 2 dx = n

ρ0π
l

10
3
( )
× 2550
= n ρ πl 3
3= 40 0 0

4 12. (b) : The given system can be considered to 7. (d) : Emf developed across the given edges, = Blv = 0.1 × 0.02 × 6 = 12 × 10–3 V

be a parallel combination of three capacitors C1, C2 and C3. As all the edges are parallel between the faces The equivalent capacitance
C3 perpendicular to the x-axis, hence required

Hence, potential difference is 12 mV. 8. (c) : The normal reaction N


1= k 1 ε 0A C = k 2 ε 0A
C 3 d , 2 3 d ,
C = k 3ε 0A ⎛⎜
3 3 d on the block N = m ⎝g + 2
g ⎞⎟ = ε
⎠ 3mg K ′ 0A

= K 1+ K 3
ε ⎞⎟
2+ K 3 ⎛⎜ ⎝ 0d A ⎠2 The distanc
K K
Hence, K = 1 23

K
3

1 0 1 ⎛⎜
system in time t is S = ut + 2 at = + 2 ⎝2
g ⎞⎟ =g ++ + +
⎠t t = 10 12 3 14
12
= 13. (a) : The efficiency of all the eng

i.e., 1 = 2 = 3 2 2 So, the work done by normal reaction on the


⎛⎜ 3
1 21 1 1 block is F N ⋅ S = ⎝ 2
⎞⎟ ⎛⎜ g ⎞⎟ = T
mg ⎠ ⎝ t ⎠ mg t − T

= − TT 3 2

= − TT 43 TT232 2 = = TT12TT

T2T3 = T1T4 ...(iii) 38

Downloaded from jeemain.guru


22
= ⎛⎜
T 22 T1
TT 1 4 (Using (i) and (iii))
⎝ T 2 ⎞⎟ ⎠ Similarly, T2 = (TT3 21T= 4)(T1/3

1T 42)1/3 (Using (ii) and (iii)) 14. (d) : As the object of mass m is ejected from the satellite, which has same speed v so its total energy i

mv 2 .
1 mv2
In potential energy of object m while in satellite = –(2 × K.E.) = –2 × 2 = –mv2 Let K.E. given to mass m = x. So total energy =
1
[Total energy at infinity is zero] x = mv2 Hence at the time of ejection, the kinetic energy of the object is 2

mv 2 + 12

mv 2 = mv 2 .
10 − 10
15. (b) : In the steady state, the energy flux is K ΔTΔ x= 0 . 1 × ( 3 1
90
2) =

W m 2 16. (c, d) at distance : The potential r is given as

4 π at p G ∈ . 0 a r G r poin
For a point at which the potential is equal as
A

RBPXr4 4

qaπε 0 r

2= 4 πε 0 2

( qar − R )
2
2 1
or r 2= ( r −
R
R ) 2 or ± 2( r − R ) = r Hence, 2( r − R ) = r or r = 1( 2 −2 ) Als
17. (d) : Consider a strip of radius x and thickness dx. Reaction force by this strip Friction force on the strip,
F 2( π xdx )
=π R2
=μ F 2( xdx )
f R 2 Torque to overcome the friction,
dxx
fx μ F
Rτ =∫ 0 = R 2

R ∫ 2( x 2
dx

)0
μ F
= R 2

⎡⎢
× ⎢
⎣ x3
R
⎤⎥⎥ ⎦ 0

=2
μ
3 FR 18. R G × (c) F G : Torque = I α due to force F, R G × F G
α=3
2
F
=v
FMR 19. (b) : Range, R
02
2sin g θ A1A 2

π 22
R R12 max max

= vv 1244 = 16 120. (a) : The minimum energy


=
E 2
p = ⎛⎜
m2 ⎝h
2

λ
⎞⎟ ×
⎠12 m = ⎛⎜ ⎝66 . 75 . ×
10 10
−− 3412

⎞⎟ ⎠2

1 2 × 91 . × 10 − 31 × 16 .

×
10

19
25 keV 21. (d) = 100 × : The maximum electric field, 10–6 × 3 × 108 = 3 × 104 N C–1
E0 = B0c
22. (c) : Let q1, q2 and q3 be the charge on the spherical shell with radii a, b and c respectively. The surface charge density at the three
q
So, a
=
2 22q 3

4 π 1 2 = 4 π qb
= 4 qπ 3c
q c q a q =
2 or 1 b 22 b 2 2 2 Also, q1 + q2 + q3 = Q ab 22 q 2 q 2
c qQ
b
=
2 2 + 2 + 2 Charge q
b Q =
b a c 2⎛2

2
⎜ ⎝2 + 2 +
c
2 ⎞⎟ ⎠q b
bQ1
=
bac 2⎛2

⎜ ⎝2 + 2 +

2 ⎞⎟ ⎠The potential at a point at distance r from common centre is


3
bQ
bac
Downloaded from jeemain.guru
23 Solved Paper 2019
kq
V= a
1
+ kqb
2
+ kqc 3
kq
= 2

⎛⎜
⎝27. (b) : Since height of water in the given large
tank remains same so
a
b 22
a1+b
1 + b c22

1 c ⎞⎟ ⎠= 4 πε
1
0
water inflow rate = water outflow rate
Q i = Av =

2+ 2+

2b Q++acb
2a
bcb
= i 22 2

= 10 −
8 10 × = 4 πε
+ +
Q 0a 2a + bb2 +
c c 2−8
2 × 98
.=

19 1. 6 m
= 0.051 m = 5.1 cm
28. (d) : Consider the circuit I2 23. (d) : Let l be the length of wire AJ at which galvanometer shows zero deflection.
Applying KVL on lower loop,
I1 1 R1 2 IR

εε
10V 10V r r
R2

+
12
⎛⎜ ⎝12

L rl Apply KVL for both the loops


I1R1 – 10 = 0 I1 = 1020 = 0 . 5 A ...(i) – 10 + I1R1 + I2R2 = 0 or or II 2 1 = + 0 I 2 A = 1020 = 0 . 5 A
[Using (i)]
29. (a) : Output R = ( X + Y ) + ( YX . )
= ( X + Y )( YX . ) = ( XY )( YX
. ) XX X + Y Y Y
⎞⎟ = l 13 L 1 25 10 − 3
PQ ⎠ 2 = 24 24. (b) : Path difference = dsin d × = ( 01 . mm ) 40 = .× mm = 2500 nm For bright fringes, path
So, 2500 = n 1 = m 2 or 1 = 500 nm, 2 = 625 nm 25. (a) : The resistance of the resistor is 50 × 102 . So, the maximum current that can b
it is
2
PR = 50 ×
A 20 mA
10 2 = 26. (a) : According to law of radioactivity, the count rate at t = 8 seconds is

N1 = N0 e– t dNdt 1600 = λ N = λ N 0
e − λ t = N0 e0 = N0 100 = N0 e–8 = 1600 e–8 e8 = 16 = 24 e2 = 2 At t = 6 sec

dNdt = = 1600 N0e–6 = 1600 × (e–2 )3


1 = 200.
× 8
R
Y X. So, for output R = 1, X = 1, Y = 0 30. (c) : The velocity of wave in the string is
Tm l/ . So, the distance between successive nodes is
λ 2
= 12
υv=

2 1υ Tm l/
=2×
1 100

1 5 × 10 −
3

= 0.2 m = 20 cm
Downloaded from jeemain.guru
24 JEE Main

JEE MAIN
Held on Solved Pape
2019
10th January 2nd SHIFT 1.
An unknown metal of mass 192 g heated to
a temperature of 100°C was immersed into a brass calorimeter of mass 128 g containing 240 g of water at a temperature of 8.4°C. Calcu
heat of the unknown metal if water temperature stabilizes at 21.5°C. (Specific heat of brass is 394 J kg –1 K–1) (a) 485 J kg–1 K–1 (b) 916
654 J kg–1 K–1 (d) 1232 J kg–1 K–1 2. Two kg of a monoatomic gas is at a pressure of 4 × 10 4 N/m2. The density of the gas is 8 kg/m3. W
of energy of the gas due to its thermal motion? (a) 104 J (b) 103 J (c) 106 J (d) 105 J 3. The modulation frequency of an AM radio station
which is 10% of the carrier wave. If another AM station approaches you for license, what broadcast frequency will you allot? (a) 2750
kHz (c) 2250 kHz (d) 2000 kHz 4. The self induced emf of a coil is 25 volts. When the current in it is changed at uniform rate from 10
the change in the energy of the inductance is (a) 637.5 J (b) 540 J (c) 437.5 J (d) 740 J 5. The eye can be regarded as a single refracting
radius of curvature of this surface is equal to that of cornea (7.8 mm). This surface separates two media of refractive indices 1 and 1.34
distance from the refracting surface at which a parallel beam of light will come to focus (a) 4.0 cm (b) 1 cm (c) 3.1 cm (d) 2 cm
6. The actual value of resistance R, shown in the figure is 30 . This is measured in an experiment as shown using the standard formul
= V , where V and I are the reading
R I and ammeter, respectively. If the measured value of R is 5% less, then the internal resistanc

(a) 570 (b) 600 (c) 35 (d) 350


7. Consider the nuclear fission
Ne 20 ⎯ →⎯ 2 He 4 + C 12 Given that the binding energy / nucleon of Ne20, He4 and C12 are, respectively, 8.03 MeV, 7.07 MeV and 7
identify the correct statement. (a) energy of 11.9 MeV has to be supplied (b) energy of 12.4 MeV will be supplied (c) 8.3 MeV energy w
(d) energy of 3.6 MeV will be released
8. A parallel plate capacitor having capacitance 12 pF is charged by a battery to a potential difference of 10 V between its plates. The c
now disconnected and a porcelain slab of dielectric constant 6.5 is slipped between the plates. The work done by the capacitor on the sl
(b) 692 pJ (c) 560 pJ (d) 600 pJ
Downloaded from jeemain.guru
25 Solved Paper 2019
9. A particle starts from the origin at time t = 0 and moves along the positive x-axis. The graph of velocity with respect to time is show
is the position of the particle at time t = 5 s ?
(a) 9 m (b) 6 m (c) 10 m (d) 3 m 10. Two forces P and Q, of magnitude 2F and 3F, respectively, are at an angle with each other. If the
doubled, then their resultant also gets doubled. Then, the angle is (a) 120° (b) 60° (c) 90° (d) 30° 11. Two identical spherical balls of m
R each are stuck on two ends of a rod of length 2R and mass M(see figure). The moment of inertia of the system about the axis passing
through the centre of the rod is
209
(a) 15
G
(a) Brown, Blue, Brown (b) Brown, Blue, Black (c) Grey, Black, Brown (d) Red, Green, Brown 13. A by d G particle = F 4 iG=
G
12 j , is undergoes experiencing If the particle has a force, given a displacement a kinetic energy of 3 J at the beginning of the disp
its kinetic energy at the end of the displacement ? (a) 9 J (b) 12 J (c) 10 J (d) 15 J 14. A particle executes simple harmonic motion with
5 cm. When the particle is at 4 cm from the mean position, the magnitude of its velocity in SI units is equal to that of its acceleration. T
8π 4π 3π 7π
time in second is (a) 3(b) 3(c) 8(d) 315. A rigid massless rod of length 3l has two masses attached at each end as show
The rod is pivoted at point P on the horizontal axis (see figure). When released from initial horizontal position, its instantaneous angula
will be
g (b) g (c) 7g (d) g
(a) l3 l2 3l 13 l MR 2 (b) 137
15

MR
2 16. For the circuit shown below, the current
152
through the Zener diode is (c) 15

MR 2 (d) 1715

MR 2 12. The Wheatstone bridge


shown in figure here gets balanced when the carbon resistor used as R1 has the colour code
(a) Zero (b) 5 mA (c) 9 mA (d) 14 mA (Oran
17. A hoop and a solid cylinder of same mass and The resistor R2 and
radius are made of a permanent magnetic R4 are 80 and 40 ,
material with their magnetic moment parallel respectively. Assume that the colour code
to their respective axes. But the magnetic for the carbon resistors gives their accurate
moment of hoop is twice of solid cylinder. values, the colour code for the carbon resistor,
They are placed in a uniform magnetic used as R3, would be
field in such a manner that their magnetic
Downloaded from jeemain.guru
26
moments make a small angle with the field. If the oscillation periods of hoop and cylinder are Th and Tc respectively, then (a) Th = 0.5T
–4 2 2
(c) Th = 1.5Tc (d) Th = Tc 18. A metal plate of area 1 × 10 m is illuminated by a radiation of intensity 16 mW/m . The work function
eV. The energy of the incident photons is 10 eV and only 10% of it produces photo electrons. The number of emitted photo electrons p
their maximum energy, respectively, will be [1 eV = 1.6 × 10 –19 J] (a) 1010 and 5 eV (b) 1012 and 5 eV (c) 1011 and 5 eV (d) 1014 and 10
closed organ pipe has a fundamental frequency of 1.5 kHz. The number of overtones that can be distinctly heard by a person with this o
(Assume that the highest frequency a person can hear is 20,000 Hz) (a) 5 (b) 6 (c) 4 (d) 7 20. Two stars of masses 3 × 1031 kg each, and
1011 m rotate in a plane about their common centre of mass O. A meteorite passes through O moving perpendicular to the star’s rotatio
to escape from the gravitational field of this double star, the minimum speed that meteorite should have at O is (Take Gravitational con
10–11 N m2 kg–2) (a) 2.8 × 105 m/s (b) 1.4 × 105 m/s (c) 3.8 × 104 m/s (d) 2.4 × 104 m/s
21. A current of 2 mA was passed through an unknown resistor which dissipated a power of 4.4 W. Dissipated power when an ideal po
V is connected across it is (a) 11 × 10–4 W (b) 11 × 10–5 W (c) 11 × 105 W (d) 11 × 10–3 W
22. The electric field of a plane polarized electromagnetic wave in free space at time t = 0 is given by an expression GEx y j x z ( , ) c
G
ˆ [ ] The magnetic field Bxzt ( , , ) is given by (c is the velocity of light)
1 ˆˆ 1 ˆˆ 1
(a) 6 8 6 8 10 c k i x z ct ( )cos ( ) [ ] − + − (b) 6 8 6 8 10 c k i x z ct ( )cos ( ) [ ] + + − (c) 6 8 6 8 10 c k i x z ct ( )cos ( )
1 ˆˆ
6 8 6 8 10 c k i x z ct ( )cos ( ) [ ] + − + 23. Half mole of an ideal monoatomic gas is heated at constant pressure of 1 atm from 20
Work done by gas is close to (Gas constant R = 8.31 J/mol. K) (a) 291 J (b) 73 J (c) 581 J (d) 146 J 24. Charges –q and +q located at A
respectively, constitute an electric dipole. Distance AB = 2a, O is the mid point of the dipole and OP is perpendicular to AB. A charge

where OP = y and y > > 2a. The charge Q experiences an electrostatic force F. If Q is now moved along the equatorial line to P such t
⎛ ⎜ ⎞ ⎟ , force on Q will be y ⎛ ⎜⎞ ⎟ F (d) 9F
⎝ ⎠ 3 close to a 3 2 >> ⎝ ⎠ (a) 27F (b) 3F (c) 3 25. Consider a Young’s double s
shown in figure. What should be the slit separation d in terms of wavelength such that the first minima occurs directly in front of the sli
(a)
λ
(b) λ
2 5 2( ) −

( )5 2 − (c) λ
(d) λ
( )5 2−
2 5 2( ) − 26. Four equal point charges Q each are placed in the xy plane at (0, 2), (4, 2), (4, –2) and (0, –2). The work required to put
at the origin of the coordinate system will be
Downloaded from jeemain.guru
27 Solved Paper 2019
Q24 ⎛⎜ 1 + 1 ⎞⎟ (b) Q2
(a) πε 0 ⎝ 5 ⎠
2 2πε Q2 4 ⎛⎜ 1 + 1 ⎞⎟ (d) Q24πε
0 (c) πε 0 ⎝ 3 ⎠ 0 27. A cylindrical plastic bottle of negligible mass is filled with 310 ml of w
floating in a pond with still water. If pressed downward slightly and released, it starts performing simple harmonic motion at angular fr
radius of the bottle is 2.5 cm then is close to (density of water = 103 kg/m3) (a) 2.50 rad s–1 (b) 3.75 rad s–1 (c) 5.00 rad 28. Two vectors

magnitude A G and B G is
G –1 G G G G G
of sA and ( A of −B (B A have ). + (d) The B equal ) 1.25 rad s–1

magnitudes. is ‘n’ times the angle bet


−1 −
(c) sin ⎡ ⎢ ⎣ nn +
1 ⎤ ⎥
1 ⎦ (d) sin− 1 ⎡ ⎢ ⎢
2
⎣ n n2

+

11 ⎤ ⎥ ⎥ ⎦ 29. At some location on earth the horizontal component of earth’s magnetic field is 18 × 10 –6 T. At this location, magnetic
0.12 m and pole strength 1.8 A m is suspended from its mid-point using a thread, it makes 45° angle with horizontal in equilibrium. To
horizontal, the vertical force that should be applied at one of its ends is (a) 1.8 × 10–5 N (b) 3.6 × 10–5 N (c) 6.5 × 10–5 N (d) 1.3
30. The diameter and height of a cylinder are measured by a meter scale to be 12.6 ± 0.1 cm and 34.2 ± 0.1 cm, respectively. What will
2
its volume in appropriate 1 ⎡ ⎢ ⎢ ⎣ n n 2
significant figures ? (a) 4264 ± 81 cm3 (b) 4260 ± 80 cm3 (c) 4300 ± 80 cm3 (d) 4264.4 ± 81.0 cm3

SOLUTIONS
1. (b) : Heat lost by unknown metal = Heat gained by calorimeter and water 192 × S × (100 – 21.5) = 128 × (394) × (21.5 – 8.4) + 240
8.4) 192 × S × 78.5 = 13.1 (394 × 128 + 240 × 4200) S
= ⎛ N ⎞ = ⎛ RT ⎞
2. (a) : Thermal energy of N molecules of a monoatomic gas, E N ⎝ kT NA⎤ ⎥ ⎥ ⎦ ⎠ ⎝ ⎠ 11
−1
(b) cos ⎡ ⎢ ⎣ nn

+
1 ⎤ ⎥
1 ⎦ 10% of 2000 kHz = 200 kHz
L Δ I or 25 L −⎛ 2
Hence allocated broadcast frequency will be 2000 kHz 4. (c) : = 25 V, I1 = 10 A, I2 = 25 A t = 1 s, E = ? ε = Δ t = ⎝
⎞ ;L=
⎠ 15
25= 1
53 H 33ΔE = 2

L ( I 22 − I 12 ) = 12

× 53 × ( 25 2 − 10 2 ) 2

2
=2
3
( nRT ) = 32
⎛⎜ 5 525 437 5. J
PV = 2 3P ⎝= 6 × =
m ⎞⎟ 3 4 10 4 2 15 10 4
5. (c) : ρ ⎠= 2 × × × 8= .× J
250
3. (d) : 10% of c = 250 kHz υc =
100 2500 kHz
10 × = Hence, range of signal = (2500 250 kHz) = 2250 kHz to 2750 kHz
R = 7.8 mm C u =
= 1 = 1.34
1 2

v=f
μ μ μ −μ 1 . 34 − μ
Using refraction formula, v 2− u 1= 2R 1 or f
=1.
∞1
34
7.
8
−1

=3.
478

Downloaded from jeemain.guru


.
28f = 1
34 3 . × 4 78
30 . 7 mm 3.1 cm
= ≈
R R
6. (a) : Ra = 30 , Rm = Ra – As, 0.95 Ra = a

( +
v Ra
R
)5 ×R = 0.95 R 20 4 12
v 100 a a 0.95 0.95 Rv = RR19 a a + = Ra 0.95 0.05 = 19 RR× v v = Rv 30 = 570 7. (a) : Ne 2He + C Q = (20 – valu
= ((2 (BE)× initial 7.07 – × (BE)4) + final 7.86 × 12) = 9.72 MeV 8. (a) : Work done = Change in energy stored in
Q
W=2 C 21

2 QC 22 the system Q C2 = = C6.5 1V C= 1 (12 pF) = 6.5 × (10 12 × V) 10= –12 120 F
=Q ⎛⎜ − ⎞⎟
× 10–12 C W C 2 21 ⎝
11 ⎛⎜ 1
⎠ 6 5. = C 1 2 V2 ⎝ − 13
2⎞⎟ = × ×
⎠ 100 12 2 10
− 12
⎛⎜ 1
⎝ − 13
2⎞⎟ 1 2 2 2 2
⎠508 pJ 9. (a) : As particle moves in straight line. x(t = 5) = Area under the graph between t = 0 to t = 5 s = 2 × × + ×
10. (a) : Resultant of two forces is given by
R = P 2 + Q 2 + 2 PQ cosθ ...(i)
and R ′ = P 2 + 4 Q 2 + 4 PQ cosθ
Also, 2R = R or 4R2 = R 2 4(P3P2 2 + Q2 + 2PQ cos + 4PQ cos = 0
) = (P2 + 4Q2 + 4PQ cos )
12
12F2 + 4(6F2) cos = 0 or cosθ=− 24
1 2 2⎡ ⎢ 2 2 ⎤⎥ MR2 ⎛ 4 ⎞
=− 12 = 120° 11. (b) : I = Irod + 2 × Iball = 12 M ( R)2+ ⎣ 5 MR 2 + M ( R)2 ⎦= ⎝ 12 + 445 ⎠

MR
2
Orange Red Brown

12. (a) : R1 = 32 × 101 = 320 For balanced Wheatstone bridge, RR 12


=
R R3 4
or
R3
320 =
80 40 R3 = 160 = 16 × 10
G G
Brown Blue Brown 13. (d) : Using work energy theorem, Work ( F G 3 ⋅ i d G − = 12 done K jf)⋅ −
G 2
( by 4 K i i ) all = Kforces f = Change in KE − 3 12 – 0 = Kf – 3 Kf = 15 J 14. (a) : In SHM, speed, v = A − x 2 Acceleration, a = –
A2
ω A 2 − x 2 = ω 2 x A 2 − x 2 = ω 2 x 2 ω2 = x

2

x
2


= 5 24 2

4
⎛ ⎞ 3 T=2
2= ⎝ 32 4 ⎠ω= 4 ω
=8
π 3π
g
15. (d) : = I 5M0 g l – 2M0 g (2l) = [5M0 l2 + 2M0(2l)2] M0 g l = = 13M 13
120
l 0 l2 16. (c) : Assume zener diode does not undergo breakdown. Current in the circuit, I = 15000
8 10 − 3 8
= × A= mA
So voltage drop across diode = (10 k ) (8 mA)
= 80 V > 50 V Hence, the diode undergoes breakdown.
120 V
R1 = 5 k
70 V
50 V R2 = 10 k

Downloaded from jeemain.guru


29 Solved Paper 2019
70
Current in R1, I1 =
= 14 mA 50
5000 Current in R2, I2 =
= 5 mA
10000 Current through zener diode = 14 – 5 = 9 mA
17. (d) : Time period of a oscillating dipole,
= 2π I MR T
T μ BT B

22. (a) : Ex ( , y ) = 10 j cos( 6 x + 8 z ) E = 10 j cos[( 6 i + 8 k ) ⋅ ( xi + zk )] = 10j cos[ K ⋅ r ] K = 6 i + 8 k ; This is direction of w


direction of ‘C’
l
Direction of B will be along
−4i+3k G
C × E = Magnitude 5 of B will be along Magnitude
−4i+3k
cC×E= 5

Bc
=
E ( )10 j 2 h 2π(2μ),
=
c

2
π ⎛⎝ 12 ⎞⎠ MR2

μBB
G3 i + 4k 5

= C Hence, Th = Tc 18. (c) : = 5 eV, E = 10 eV


KE = E – = 10 – 5 = 5 eV Energy incident on the plate, E = (16 × 10–3) × (1 × 10–4) J/s = 16 × 10
= 10 ⎛⎜ − 4
⎝ i5
+3

⎞⎟ = − +
k ⎠ ( 8ic 6k)
Number of photons emitted per second
′E = 0.1 × 10 13
23. (a) : n = 0.5, T = 70°C = 70 K = 10% of E 10 eV/s
= 1011
eV s–1 19. (d) : Fundamental frequency of the closed organ pipe, 0 = 1.5 kHz = 1500 Hz Resonant frequency is odd multiple of
Work done, W = P V = nR T = 0.5 × 8.31 × 70 291 J 24. (a) : Electric field due to dipole on equatorial
40 or n < 6.16
(2n + 1) 0 20000 or (2n + 1) 1500 20000 or (2n + 1) 3 Possible values of n are 0, 1, 2, 3, 4, 5 and 6.
=− p
plane, E G At At k r G 3 point P, point P , F
Q G =− ⎛⎜
3 F P′

− pGQ GMm
⎝ ky 20. (a) : Using energy conservation principle at points O and (TE)at O = (TE)at or −
r
− + = + Q (0, –2)
...(i)

...(ii)
⎞( )/3 3 ⎟ ⎠ From F G P ′ = equation 27 F G P = 27 (i) F G and (ii), 25. (a) : Path difference, S2P
1 mv 2
r 2
00
422
= 4 GM × ×
v r=4 667 10 − 10
× ×
3 10

d 2+ d 2− d = λ
.
11 31
11
P
= 2.8 × 105 m/s 21. (b) : Case I : As I2 R = P ; R = I2 R = × −

= × d( )5 − 2 = λ 2 d = 2 2( 5 λ −) 26. (a) : Required work = change in potential energy =


(4, 2) ⎛⎜
yQ (0, 2) Q ⎝+ 12

+ 120 + 120 4 .
4 ( 2 10 3 ) 2
11 . 10 Ω
6

⎞⎟ −

=V Q24 πε ⎛⎜ 1 + 1 ⎞⎟
0 Q Case II : P R= 0 ⎝ 5
⎠2 (4, –2)
=
×2
6
=×− x(
Q 11
) 11 10 W
11 . 10 5

Downloaded from jeemain.guru


⎛⎜ ρAg ⎞⎟ x ω ρ
3027. (*) : Restoring force due to pressing the bottle with small amount x, F = –( Ax)g a = – ⎝ m ⎠ 2= m
Ag ρ π 2
=
(r)
gm
×π× ×
ω = 10 3 (2.5 10 −
2)
2
×

10 310 ×
10

3
G G G
7.95 rad/s * None of 28. (a) : the A + given B = nA option −
G
B is correct. A2 + B2 + 2AB cos = n2(A2 + B2 – 2AB cos ) 2AB cos (1 + n2) = (A2 + B2) (n2 – 1) 2A2 cos (1 + n2) = 2A2 (n2 – 1)
n 2
= cos–1 ⎡ ⎢ ⎣ n 2

−+
⎤⎥
11 ⎦

29. (c) : BH = 18 × 10–6 T, m = 1.8 A m, l = 0.12 m


=
F FB

B l
H F × 2 sin 45 °= μ B sin 45 ° F = 2 B/l
π D 2l
45° μ GFG= 2mB = 2 × 1.8 × 18 × 10–6 = 6.48 × 10–5 N 30. (b) : D = 12.6 0.1 cm, h = 34.2 0.1 cm V = 4

= 14
××2
× 227 ( 12 . 6 ) ( 34 . 2 ) 4266 cm3 4260 cm3 (Three significant figures) Δ VV
=2
⎛ 2 0.
× Δ D D+ Δ l lΔV = ⎝ × 12 .
1
6

+ 34 0 .. 1
⎞ × 4260 ≈ 80
2 ⎠ cm 3 Volume = 4260 80 cm3

Downloaded from jeemain.guru


31 Solved Paper 2019
1. A hydrogen atom, initially in the ground state, is excited by absorbing a photon of wavelength 980 Å. The radius of the atom in th
terms will be (hc = 12500 eV-Å)
of Bohr radius a0,
(a) 9a0 (b) 16a0 (c) 4a0 (d) 25a0 2. The variation of refractive index of a crown glass thin prism with wavelength of the incident light
of the following the angle of graphs minimum
correct one, if Dm is
(a)
(b)
(c)
(d)

JEE MAIN
Held on Solved Pape
2019
11th January 1st SHIFT 3.
Two equal resistances when connected in series to a battery, consume electric power of 60 W. If these resistance
connected in parallel combination to the same battery, the electric power consumed will be (a) 240 W (b) 120 W (c) 60 W (d) 30 W 4.
distance of 20 m from a convex lens of focal length 0.3 m. The lens forms an image of the object. If the object moves away from the len
m/s, the speed and direction of the image will be (a) 2.26 × 10 –3 m/s away from the lens (b) 3.22 × 10–3 m/s towards the lens (c) 1.16 ×
the lens (d) 0.92 × 10–3 m/s away from the lens 5. The resistance of the meter bridge AB in given figure is 4 . With a cell of emf = 0.5 V
resistance Rh = 2 the null point is obtained at some point J. When the cell is replaced by another one of emf = the same null The emf 2

point J is found for Rh = 6 2 .


(a) 0.5 V (b) 0.4 V (c) 0.6 V (d) 0.3 V 6. An equilateral triangle ABC is cut from a thin solid sheet of wood. (See figure) D, E and F ar
of its sides as shown and G is the centre of
Downloaded from jeemain.guru
32
the triangle. The moment of inertia of the triangle about an axis passing through G and perpendicular to the plane of the triangle is I0. If
15 (b) I
triangle DEF is removed from ABC, the moment of inertia of the remaining figure about the same axis is I. Then (a) I = 16 I 0
3 (d) I I
4I0 = 4 07. In an experiment, electrons are accelerated, from rest, by applying a voltage of 500 V. Calculate the radius of the p
magnetic field 100 mT is then applied. [Charge of the electron = 1.6 × 10 –19 C, Mass of the electron = 9.1 × 10–31 kg] (a) 7.5 m (b) 7.5
× 10–4 m (d) 7.5 × 10–3 m 8. In the figure shown below, the charge on the left plate of the 10 F capacitor is –30 C. The charge on the rig
F capacitor is (a) –18 C (b) –12 C (c) +12 C (d) +18 C
9. Three charges Q, +q and +q are placed at the vertices of a right-angle isosceles triangle as shown below. The net electrostatic energy
configuration is zero, if the value of Q is (a) +q (b) –2q

(c)
2q −
2 +1(d) +
q12
10. Ice at –20°C is added to 50 g of water at 40°C. When the temperature of the mixture reaches 0°C, it is found that 20 g of ice is still
amount of ice added to the water was close to (Specific heat of water = 4.2 J/g/°C Specific heat of Ice = 2.1 J/g/°C Heat of fusion of wa
J/g)
(a) 60 g (b) 50 g (c) 40 g (d) 100 g
11. A satellite is revolving in a circular orbit at a height h from the earth surface, such that h < < R where R is the radius of the earth. A
effect of earth’s atmosphere can be neglected the minimum increase in the speed required so that the satellite could escape from the gra
earth is (a) 2gR (b) gR( )2 1−
gR (d) gR
(c) 2 12. The given graph shows variation (with
distance r from centre) of
(a) Electric field of a uniformly charged
spherical shell (b) Electric field of a uniformly charged sphere (c) Potential of a uniformly charged spherical
G G
shell (d) Potential of a uniformly charged sphere 13. A slab is subjected to two forces F1 and F2 of same magnitude F as shown in
G ˆˆ
Force F2 is in XY-plane while force F1 acts along z-axis at the point ( ). 2 3 i j + The moment of these forces about point O will be
ˆˆˆ ˆˆˆ ˆˆˆ ˆˆˆ
(a) ( ) 3 2 3 i j k F − − (b) ( ) 3 2 3 i j k F + + (c) ( ) 3 2 3 i j k F − + (d) ( ) 3 2 3 i j k F + − 14. Equation of travellin
stretched string of linear density 5 g/m is y = 0.03 sin(450t – 9x) where distance and time are measured in SI units. The tension in the st
(b) 7.5 N (c) 5 N (d) 12.5 N
Downloaded from jeemain.guru
33 Solved Paper 2019
15. An amplitude modulated signal is given by V(t) = 10[1 + 0.3 cos(2.2 × 104t)] sin(5.5 × 105t). Here t is in seconds. The side band fre
kHz) are, [Given = 22/7] (a) 178.5 and 171.5 (b) 89.25 and 85.75 (c) 1785 and 1715 (d) 892.5 and 857.5 16. A rigid diatomic ideal gas
2
adiabatic process at room temperature. The relation between temperature and volume for this process is TV x = constant, then x is (a)
(d) 5
3 17. A particle is moving along a circular path with a constant speed of 10 m s –1. What is the magnitude of the change in velocity
when it moves through an angle of 60° around the centre of the circle? (a) zero (b) 10 2 m/s (c) 10 3 m/s (d) 10 m/s 18. In a Wheatston
fig), Resistances P and Q are approximately equal. When R = 400 , the bridge is balanced. On interchanging P and Q, the value of R, f
405 , The value of X is close to (a) 401.5 ohm (b) 404.5 ohm (c) 403.5 ohm (d) 402.5 ohm 19. A body is projected at t = 0 with a veloc
angle of 60° with the horizontal. The radius of curvature of its trajectory at t = 1 s is R. Neglecting air resistance and taking acceleration
= 10 m s–2, the value of R is (a) 2.5 m (b) 2.8 m (c) 10.3 m (d) 5.1 m 20. In the circuit shown,
the switch S1 is closed at time t = 0 and the

switch S2 is kept open. At some later time(t0), the switch S1 is opened and S2 is closed. The behavior of the current I as a function o
by
(a) (b)
(c) (d)
21. A body of mass 1 kg falls freely from a height of 100 m, on a platform of mass 3 kg which is mounted on a spring having spring co
106 N/m. The body sticks to the platform and the spring’s maximum compression is found to be x. Given that g = 10 m s–2, the value
to (a) 8 cm (b) 40 cm (c) 80 cm (d) 4 cm 22. In a Young’s double slit experiment, the path difference, at a certain point on the scr

interfering waves is wavelength. The ratio of the intensi


point to that at the centre of a bright fringe is close to (a) 0.80 (b) 0.94 (c) 0.85 (d) 0.74 23. The force of interaction between two atoms
= αβ exp ⎛⎜ − x
given by F ⎝ α kT 2
⎞⎟ ;

where x is the
distance, k is the Boltzmann constant and T is temperature and and are two constants. The dimension of is (a) (c) MMLT 2L2T –2 –2 (b)
2
T –4
–4

24. In the given circuit the current through Zener


Diode is close to
(a) 4.0 mA (b) 0.0 mA (c) 6.0 mA (d) 6.7 mA 25. An electromagnetic wave of intensity 50 W m–2 enters in a medium of refractive inde
Downloaded from jeemain.guru
34
without any loss. The ratio of the magnitudes of electric fields, and the ratio of the magnitudes of magnetic fields of the wave before
into the medium are respectively, given by
⎛⎜ 1 ⎞⎟ (b) ( n , n ) ⎛⎜ 1 , 1 ⎞⎟ (d) ⎛⎜ 1 , n ⎞⎟
(a) ⎝n , n ⎠ (c) ⎝ n n ⎠ ⎝ n ⎠26. If the deBroglie wavelength of an ele
–3
10 times the wavelength of a photon of frequency 6 × 10 Hz, then the speed of electron is equal to (Speed of light = 3 × 10 8 m/s Pla
14

6.63 × 10–34 Js Mass of electron = 9.1 × 10–31 kg) (a) 1.7 × 106 m/s (b) 1.8 × 106 m/s (c) 1.45 × 106 m/s (d) 1.1 × 106 m
27. A liquid of density is coming out of a hose pipe of radius a with horizontal speed v and hits a mesh. 50% of the liquid passes
unaffected. 25% looses all of its momentum and 25% comes back with the same speed. The resultant pressure on the mesh will be
n
⋅ 12 (d) 1n2 30. A gas mixture consists of 3 moles of oxygen and 5 moles of argon at temperature T. Considering only translationa

modes, the total internal energy of the system is (a) 20 RT (b) 12 RT (c) 4 RT (d) 15 RT

SOLUTIONS
hc
1. (b) : The energy absorbed So,
= − ⎛⎜
λ
− hc
⎝n n by the atom is λ
1
(a) v2 (b) 4
πt The ratio o
ρv 2 28. A particle undergoing simple harmonic motion has time dependent displacement given by x ( t ) = A sin 90 .
1 (b) 2 (c) 1 (d) 3
potential energy of this particle at t = 210 s will be (a) 9 29. There are two long co-axial solenoids of same length l. T
outer coils have radii r1 and r2 and number of turns per unit length n1 and n2 respectively. The ratio of mutual inductance to the self indu
n2
inner- coil is (a) n1 2 1
(b) n 2 n
n 2 (c) n
2r1

rr 12 r
13 .

61 2 1
2 ⎞

2
1 ⎟ ⎠V2 R
= 60 W ⇒
2 VR 2
= 120 W

when the two resistance are connected in parallel combination power consumed is 12500 980 × 13 . 6
=
(1
− ⇒n 2
1) 2 n 1 22
th 2
4 VR The radius of atom in excited n state orbit is 4is a0 a= 0n16a2. So, 0

the radius of
2. (a) : The angle of minimum deviation
Dm = ( – 1)A. In the given graph, as the wavelength increases decreases. Hence, the angle of minimum deviation also decreases. 3. (a)
consumed when two resistance are in series combination is
2/ 2 = 120 ( 2 ) = 240 W 4. (c) : From the lens equation,
= −
1f 1v u1
...(i)

=
1v 0 1. 3
+
− 120
=
197
v= 60
60 197
m

Differentiating eqn. (i),


=−
0 v
1
2d

2 ρv (c) 34

2 ρv (d) 12
Downloaded from jeemain.guru
35 Solved Paper 2019
⎛⎜
⎝197 60
⎞⎟
⎠2
dv = ( )
dt 20 12 5
––
Also, C
q–
+ q1

= 6

qdvdt = 1 . 16 × 10 − 3 m/s q2+ 5. (d) : In first case, current i1, flowing through secondary circuit i 1 RAB
=1A
+2
Now, once the balance point is achieved,
1= RAJ i1 0.5 = RAJ (1) ...(i) Similarly, in the second case,

2= RAJ i2 ε2 = 64 +

6 ++
11

=C
= C 2 q 1 = 4q 1 ...(iii)
q22 ...(ii) [' C1 and C2 are in parallel combination] q 2 C 1 6 U

106

q 1 = q ⇒ q = 53

q 1 ⇒ q 1 = 35 ( 30 ) = 18 μ C 9. (c) : Let a be the length of two equal sides of


R ⎞⎟ ε = 6R ...(ii)
AJ ⎛⎜ ⎝ ⎠ 2 10 AJ From eqn. (i) 2 = 610 × 0 . 5 = 0.3 V
Qq
the triangle. The net electrostatic k a
energy of the system is zero.
6. (a) : Moment of inertia at triangular lamina, I0 = KMa2 where K = constant of proportionality Now moment of inertia of small lamin
M a KMa
I=K 4 2 16
⎛⎜
⎝+ qa 2
0 =
+ Qq2 a Q = − 2 ⎞⎟ ⎠ 2+1 q 10. (c) : Let m be the mass of ice added to water. ⎛⎜ ⎝2 (m – 20)L + mC T1 = 50C1 T2 (m – 20)

(20) = 50 × 4.2 × (40)


m(334 + 42) = 5400 + 6680 m = 40.1 g 11. (b) : The escape speed (ve) of the satellite is
2 gR ( + h ) 2 gR ( h << R ) The orbital speed (vo) of the satellite at height h is
gR ( + h ) gR Hence, the minimum increase in speed so that the satellite could escape is
2 gR − gR = gR ( 2 − 1 ) 12. (c) : As the field inside the uniformly charged hollow sphere or spherical shell is zero, so the potential in
constant, whereas outside it varies inversely with distance. 13. (c) : The moment of force about point O is given as
r 1 × F 1 + r 2 × F 2 = ( 2 i + 3 j ) × ( Fk ) + ( 6
⎞⎟ =
j ) × [ F 2 cos 30 ( − j ) + F 2 sin 30 ( − i )] = F ( 3 i − 2 j + 3 k ) 14. (d) : The given wave is y = 0.03 sin (450 t – 9x) ⎠ 2

moment of inertia of remaining part,


15I 0
I–I= 167. (c) : The velocity acquired by the electron is given as follows :
12
2 ×
mv 2 = eV v = eVm v = 2 16 .
× ×
10 −19
500 91 . × 10 −
31
= 1 . 33 ×

10

m/s
7

when the particle enters the magnetic field its speed remains unchanged
mvr 2
= qvB
=
r
mv × ×
qB r = 16 91 . . 10 −31 133 .
×
× 10 − 19 × 100
10
7

× 10 −
3
= 7 . 56 ×

10

m
−4

8. (d) : Let q1 and q2 be the charge on 6 F and 4 F respectively.

q1 + q2 = q ...(i)

Downloaded from jeemain.guru


36Compare with standard wave equation
450 50 m/s ω
y = a sin( t – kx) The velocity of travelling v = 9= wave v =
...(i)
k
Also, velocity of wave on stretched string is
T
v= μ ...(ii) Using (i) and (ii),
T μ = 50 ⇒ T = ( 50 ) 2 × 5 × 10 − 3 T = 12.5 N
15. (b) : The equation for a amplitude modulated wave is
y = (a cos mt + A) sin ct Comparing it with given equation,
= 5.5 × 105 ω
c Hz fc = c π2
= 7 × 55 . ×10
87 5
5 22 × 2 = .

kHz
= 2.2 × 104 ω
m Hz fm = m π2

=×.×
=
× 4 . kHz So, the side band frequency,
f1 = fc – fm = 87.5 – 3.500 = 84 kHz f2 = fc + fm = 91.00 kHz 16. (c) : For an adiabatic process, TV – 1 = constant For a diatomic molecu
C
γ=C V
P
= 7 22 10
3 50
2 22
7 1 2
75 So, x = γ − 1 = 5 − = 5 17. (d) : Let the particle be moving in counter clockwise direction.
vG2 60°
G
60° v 1

The initial velocity of the particle v 1 = vj The velocity of the particle after time t is
v (cos 60 ° j − sin 60 ° i ) So, the change in velocity
v G G = v 1 3( j − i ) − j 10 j 3 i = 10 m/s
2 3( j − i )| Δv |=| v 2 − v 1 |
= 2 = 2− − =5(− 1)2+(−3)2 18. (d) : For a
400
Q P= X ...(i) After interchanging P and Q,
405
QP = X ...(ii) Multiplying both the equations,
X = R 1 R 2 = 400 × 405 = 402 5. Ω 19. (b) : At any time t, the horizontal velocity
vx = u cos Vertical velocity,
vy = (u sin ) At t = 1 s, vx = 10 cos 60° = 5 m/s
vy = ( 5 3 − 10 ) m/s The radius of curvature at time t,
v
R= a
2
=v2g
+
cos φ = v g2 x cos v

φ

25 75 100 100 3
)
10 φv
v G= 10 m s − 1 v sin 60°v G60°

g cos
g
v cos 60°
t
v
tanφ = yvx
=535

10
2.
= 15° R =
7 2 . 79 2 . 8 m
cos 15 = 20. (a, b, c *) : Initially, when is closed the current increases i the switch = i0 (1 – e–t/ S1 ) and when the
exponentially So, the behaviour of S2 is closed the current the i.e., according current can be to dep
from the following graph :
Downloaded from jeemain.guru
37 Solved Paper 2019
I
t0 t

*As per official answer key. Value t0 is not specified hence a, b and c may be correct option. 21. (*) : The moment 1 kg hits the platf
1 ( v ) + 0 = ( 1 + 3 ) v 1 ⇒ | ( 2 gh ) |= 4 v 1 v 1
gh

[ ] = [M –1T 2] Now, [ ][ ] = [F]


[M –1T 2][ ] = [ML2T –2] [ ] = [M2LT –4] 24. (b) : Net resistance of the circuit
R
R=R 1+ 2 2= 1250 Ω Current drawn from battery,
12
I = 1250
A

=
Voltage across R 2 2
I× = 1
R2 2 × 1250 2
× 1500
= 24
×
=2 10 4
×
100

= 103 8 m/s (Compression due to masses is negligible.) Using energy conservation principle,
= 7.2 V < Zener voltage (10 V) Hence, zener diode is in reverse biased without break down. So, current through diode is 0 mA.
12

Mv 12 = 12 kx 2 ;
25. (a) : The intensity of the wave remain unchanged
=M ′v
x k =
B B
So, c v
× 21 5μ 0

44
10
6
×
× 10 3
8 10
cm
4

=
2
1 μ For a non-magnetic medium, = 0, = 2 cm * None of the given options is correct.
B
BB n B n 22. (c) : The phase difference between two waves is given as
2
Δx ×
2
=8× =4
1

= ⇒ = 1 ...(i)
E c E
1 πλ πAlso, B = and 1B 1 λ

λ
v E
= ⇒ E1

BB 1 = v c= n π ESo, the intensity


cos
I = I0 2

π
8 I = I0
= =
1 nn n
[Using (i)]

26. (c) : The wavelength of photon of frequency


⎛⎜
is ⎜ ⎝1 + cos 2
π
⎞4 ⎟⎟
⎛⎜
The deBroglie wavelength of the electron is I = I 0 ⎜
⎝1 + λe =
h1
mv h2 v m ⎞2
0
⎟⎟ ⎠= .
85

I0
23. (d) : The given expression is
= αβ exp
F
⎛⎜ − x ⎞⎟ =
⎝ α kT ⎠ λ
=
e mc × h
[Given = 10–3
υ10 − 3 e ]2
αK x
×××
v=
××××
L
[
[ MLT
] [ ][ ML T K ][ K ]
]
663 .
10 −
34 6 10
14
91 .
10 − 31 3 10 8 10

3
= 1.46 × 106 m/s
α2
− −
2 2 1
=
000
27. (c) : The momentum per second carried by liquid per second is v2A.
[A is area of cross section of pipe]
Downloaded from jeemain.guru
38The force exerted due to reflected back molecules
1
2 ⎛⎜ ⎝ 4
⎞⎟
ρav 2 ⎠So, the resultant pressure is
⎛⎜
⎝12
ρ ρ
Av 2 + 14
Av
⎞⎟
2 ⎠A =

34
ρ
v 1
2 28. (*) : The maximum kinetic energy of the particle is 2

m A( 2 ω 2 ) .
1
The potential energy of the particle at any time t is 2

m ω 2 x 2 Using energy conservation


KEPE

= KEPE max − 1
KEPE

1212
ω
mA2

2= mω
2x

2−
1
=
A
2 sin 2
A

π
90 2

−1×
210

1 π
− sin 2 π 3
2 1
1 = 3 *None of the given options is correct. 29. (a) : The mutual inductance of the inner coil M = Self inductance 0n1n2 (L) r12 of th

...(i)
Using L = (i) and 0n12 (ii),

r12 ...(ii)
ML
=
⎡⎢ ⎛⎜ ⎞⎟ ⎤ ⎥
⎣ ⎝+ ⎠ ⎦ = nn 21 30. (d) : Here, oxygen molecule has three translational and two rotational degree of freedom wher
1 R
atom has three translational degree of freedom only. As each translational and rotational degree of freedom corresponds to energy 2
total internal energy of the system is
+ = + + 1
( n 1 f 1 n 2 f 2 ) RT { ( ) ( )} RT 12 33 2 53 2 =15RT

Downloaded from jeemain.guru


39 Solved Paper 2019

JEE MAIN
Held on Solved Pape
2019
11th January 2nd SHIFT 1. When 100 g of a liquid A at 100°C is added
to 50 g of a liquid B at temperature 75°C, the temperature of the mixture becomes 90°C. The temperature of the mixture, if 100 g of liq
added to 50 g of liquid B at 50°C will be (a) 70°C (b) 85°C (c) 60°C (d) 80°C
2. Two rods A and B of identical dimensions are at temperature 30°C. If A is heated upto 180°C and B upto T °C, then the new lengths
the ratio of the coefficients of linear expansion of A and B is 4 : 3, then the value of T is (a) 200°C (b) 270°C (c) 230°C (d) 250°C
3. A string is wound around a hollow cylinder of mass 5 kg and radius 0.5 m. If the string is now pulled with a horizontal force of 40 N
is rolling without slipping on a horizontal surface (see figure), then the angular acceleration of the cylinder will be (Neglect the mass an
the string) (a) 10 rad/s2 (b) 20 rad/s2 (c) 12 rad/s2 (d) 16 rad/s2
4. If speed (V), acceleration (A) and force (F) are considered as fundamental units, the dimension of Young’s modulus will be (a) V –2A
A F 2 (c) V –4A2F (d) V –4A–2F
2 2

5. A pendulum is executing simple harmonic motion and its maximum kinetic energy is
K1. If the length of the pendulum is doubled and it performs simple harmonic motion with the same amplitude as in the first case, its
energy is K2. Then
=
(a) K2 = K1 (b) K 2
K =K
2 1(c) K2 = 2K1 (d) K 2 4 16. In a double-slit experiment, green light (5303 Å) falls on a double slit having a separation of 19.
of 4.05 m. The number of bright fringes between the first and the second diffraction minima is (a) 10 (b
7. A particle of mass m is moving in a straight line with momentum p. Starting at time t = 0, a force F = kt acts in the same directi
particle during time interval T so that its momentum changes from p to 3p. Here k is a constant. The value of T is
p (b) 2k
(a) 2 k p
2p (d) 2 k
(c) k p 8. A particle of mass m and charge q is in an E G electric = 2 i ˆ + 3 j ˆ ; B and G magnetic = 4 j ˆ + 6 k ˆ . field gi

charged particle is shifted from the origin to the point P(x = 1; y = 1) along a straight path. The magnitude of the total work done is (a)
(0.35)q (c) (2.5)q (d) 5q
Downloaded from jeemain.guru
409. An amplitude modulated signal is plotted
below
Which one of the following best describes the above signal?
(a) (9 + sin(2 × 104t))sin(2.5 × 105t) V (b) (9 + sin(2.5 × 105t))sin(2 × 104t) V (c) (9 + sin(4 × 104t))sin(5 × 105t) V (d) (1 + 9sin(2
G
105t) V 10. The magnetic region field between B = Bz y ˆ. = 0 A and particle y = d of c

and charge v G= vi ˆ. If q enters d = 2 mvqB the , region with a velocity the acceleration of the charged particle at the point of its em
qvB
other side is (a) m
ˆ ˆ ⎞⎟ qvB
+⎛⎜ ⎝i 2 j ⎠(b) m ⎛⎜ ⎝2
3i j ⎞⎟ qvB 1 i j
ˆ + 12 ˆ ⎠(c) m ⎛⎜ ⎝ 2 ˆ − 32 ˆ

+⎛⎜ ⎝ˆ ˆ
ˆ ˆ ˆ ˆ
2 11. A particle moves from the point ( 2 . 0 i + 4 . 0 j ) m, at t = 0, with an initial velocity ( 50 . i + 40 . j ) m s − 1 .It is acted upo
ˆ ˆ
force which produces a constant acceleration ( 40 . i + 40 . j )ms − 2 .What is the distance of the particle from the origin at time 2 s? (
m (c) 15 m (d) 10 2 m 12. A copper wire is wound on a wooden frame, whose shape is that of an equilateral triangle. If the linear dime
side of the frame is increased by a factor of 3, keeping the number of turns of the coil per unit length of the frame the same, then the sel
the coil
⎞⎟
−ji ⎠
(a) increases by a factor of 27 (b) decreases by a factor of 9 3 (c) increases by a factor of 3 (d) decreases by a factor of 9 13. A metal ba
is heated upto 500°C and dropped into a vessel of heat capacity 800 J K–1 and containing 0.5 kg water. The initial temperature of water
30°C. What is the approximate percentage increment in the temperature of the water ? [Specific heat capacities of water and metal are,
4200 J kg–1 K–1 and 400 J kg–1 K–1] (a) 15% (b) 30% (c) 25% (d) 20% 14. The circuit shown below contains two ideal diodes, each wit
resistance of 50 . If the battery voltage is 6 V, the current through the 100 resistance (in Amperes) is
(a) 0.030 (b) 0.027 (c) 0.020 (d) 0.036 15. An electric field of 1000 V/m is applied to an electric dipole at angle of 45°. The value of ele
moment is 10–29 C m. What is the potential energy of the electric dipole? (a) –10 × 10–29 J (b) –7 × 10–27 J (c) –20 × 10–18 J (d) –9 × 10
hydrogen like atom, when an electron jumps from the M-shell to the L-shell, the wavelength of emitted radiation is . If an electron jump
25 λ (b) 27 λ 16 λ (d) 20 λ
the L-shell, the wavelength of emitted radiation will be (a) 16 20 (c) 25 27 17. A circular disc D1 of mass M
has two identical discs D2 and D3 of the same mass M and
Downloaded from jeemain.guru
41 Solved Paper 2019
radius R attached rigidly at its opposite ends (see figure). The moment of inertia of the system about the axis OO , passing through th
shown in the figure, will be
2
(a) 3MR2 (c) 3
4
(b) 5

MR
2 (a) 6 V (b) 3 V (c) 2 V (d) 1 V

MR 2 (d) MR2
18. The mass and the diameter of a planet are three times the respective values for the
23. A thermometer graduated according to a linear scale reads a value x0 when in contact with boiling water, and x0/3 when in contact
the temperature of an object Earth. The period of oscil
in °C, if this thermometer in the contact with pendulum on the Earth is 2 s. The period of oscillation of the same pendulum on the
the object reads x0/2 ? (a) 35 (b) 2
3 s
(c) 60 (d) 40 (a) 2 3s (b) 2
2 s (d) 3s
24. In the experimental set up of metre bridge shown in the figure, the null point is obtained (c) 3 2 19. In a photoelectric ex
wavelength of the light incident on a metal is changed from 300 nm to 400 nm. The decrease in the stopping potential is close to
at a distance of 40 cm from A. If a 10 resistor is connected in series with R1, the null point shifts by 10 cm. The resistance that shoul
parallel with (R1 + 10) such that the null point shifts back to its initial position is
⎛⎜ 1240 nm-V ⎞⎟
⎝hce = ⎠(a) 2.0 V (b) 0.5 V (c) 1.0 V (d) 1.5 V 20. A paramagnetic substance in the form of a cube with side
magnetic dipole moment of 20 × 10–6 J/T when a magnetic intensity of 60 × 103 A/m is applied. Its magnetic
(a) 40 (c) 60 (b) 20 (d) 30 susceptibility is (a) 3.3 × 10 –2 (b) 4.3 × 10–2 (c) 3.3 × 10–4 (d) 2.3 × 10–2
25. A galvanometer having a resistance of 20
and 30 divisions on both sides has figure of merit 0.005 ampere/division. The resistance 21. In a process, temperature a
that should be connected in series such that it mole of an ideal monoatomic gas are varied
can be used as a voltmeter upto 15 volt, is according to the relation VT = K, where K is
(a) 80 (b) 125 a constant. In this process, the temperature
2 K 1 R TΔ
(c) 120 (d) 100 of the gas is increased by T. The amount of heat absorbed by gas is (R is gas constant) (a) 3 TΔ (b) 2
26. The magnitude of torque on a particle of mass 1 kg is 2.5 N m about the origin. If the force acting on it is 1 N, and the distance of
3 R TΔ
the origin is 5 m, the angle (c) 2
between the force and the position vector is (in radians) 22. In the circuit shown, the po
between A and B is
π (b) π (c) π (d) π
(a) 3 8 4 6
Downloaded from jeemain.guru
4227. Seven capacitors, each of capacitance 2 F ,
28. A monochromatic light is incident at a certain are to be connected in a configuration to

angle on an equilateral triangular prism and obtain an effective capacitance of ⎛⎜ ⎝13


6⎞⎟ μF .
⎠ Which of the combinations, shown in figures

1. (d) : Case I : Heat lost = Heat gained 100 × sA × (100 – 90) = 50 × sB × (90 – 75) 2 sA × 10 = sB × 15 or 4 sA = 3 sB ...(i) Case II :
4
gained 100 × sA × (100 – ) = 50 × sB × ( – 50) (100 – ) = 3
suffers minimum deviation. If the refractive index of the material of the prism is 3 , then the angle of incidence is (a) 60° (b) 45° below

(c) 90° (d) 30° 29. A simple pendulum of length 1 m is


(a)
oscillating with an angular frequency 10 rad/s. The support of the pendulum starts oscillating up and down with a small angular frequen
an amplitude of 10–2 m. The relative change in the angular frequency of the pendulum is
(a) 10–5 rad/s (b) 10–1 rad/s (c) 1 rad/s (d) 10–3 rad/s 30. A 27 mW laser beam has a cr
(c)
of 10 mm2. The magnitude of the maximum electric field in this electromagnetic wave is given by [Given permittivity of space 0 c = =
speed of light
3 × 108 m/s] (a) 2 kV/m
(c) 1 kV/m (d) 1.4 kV/m

SOLUTIONS
=
3. (d) : 40 + f = ma = m(R ) ..(i) 40 × R – f × R = mR2 ...(ii) From eqn (i) and (ii), 80 = 2m R = 40 mR 40
FA
5 × 05 . = 16 rad/s2 4. (c) : Young’s modulus, Y = ll
40 N
400 = 80°C
a= 5 × 12 ( – 50)
f
2. (c) : lA = lB lOA A( T)A = lOB B( T)B ( ) ( )
/Δ / ΔTAΔT
B

= lOBl OA
×α
α F =
AB Dimensionally , Y = ( length ) 2
F2
(speed) (t
13
30 4
Speed
( )Speed
⎞⎟
Acceleration 2 ⎛⎜ ⎝ ⎠T− −= × 2 3T – 90 =
[Y ] = [F V –4 A2]
Downloaded from jeemain.guru
43 Solved Paper 2019
1 m A2 1 m v2 = 1 m A2 2 2 ...(ii)
5. (b) : K1 = K2 = 2 2 2 max 2 21 ...(i)
Here, A2 = A, From eqn. (i) and (ii)

K2K1
Hence the signal can be represented as
⎛⎜
C=
+ sin 2 π sin 2 π
⎝A a T m t ⎞⎟ ⎠ T C t = [9 + 1 sin (2 × 104 t)]
=ω ⎛⎜ ω = g
ω 1222 ⎝ l
⎞⎟ mv mv
⎠10. (*) : d = 2 qB As r =
= 2d
qB Acceleration,
v
v 1 = v 1 sin 30 ° i + v 1 cos 30 ° j = 2 i − vj y y = d

vi B GK2K 1
=
l1l 2
= ⇒ =
l12 l 1 K2
K
21
v1 sin30°

6. (c) : g = 5303 Å, d = 19.44 m a = 4.05 m


v1 cos30°

vG1

For diffraction location of first minima, y1 = Location
2D λ
aof second minima, y2 = a For interference,
dy
d sin D
O 30°
x
r

3 2
a=m
F
= qvB2 m 3( i − j ) *None of the given options is correct.

1= da
dy
λ = 4 . 80 λ Also, d sin = D
11. (a) : r 0 = ( 20 . i + 40 . j )m
2
2= da
λ = 9.62
dp or kt dt = dp
Number of bright fringes correspond 7, 8, 9. 7. (a) : As F = dt
v0
–1
= ( 50 . i + 40 . j ) m s − 1 , a = ( 40 . i + 40 . j ) m s − 2 Along x- axis, Sox = 2 m, Vox = 5 m s
to n = 5, 6,
1 a t2
ax = 4 m s–2 Sx = Sox + vox t + 2 x

= 2 + 5 × 2 + Integrating both sides,


Along y-axis,
1 ×4 × (2)2
2 = 20 m

kT ∫ 0

t dt =

∫ p

3p

= [3p
dp kT 22 T = 2 Soy = 4 m, Voy = 4 m s–1, ay = 4 m s–2

Sy = Soy + Voy t + 12 ay t2
1 × 4 × 22
=4+4×2+ 2 = 20 m
– p] = 2p
S = S xS y
p G ⎡
k 8. (d) : Work done on moving charge in a magnetic field is zero. For electric W = q V = field, q ( E ⋅ dr G ) = q ⎣ (2 i+3
⎤ = 5q
⎦ 9. (a) : Amplitude of AM wave varies as 8 V to 10 V or (9 1V) Time period of modulating = 100 s = 10 –4 s
signal, Tm
Time period of carrier signal, = 8 Tc × = 108 –6 s s
2 + 2 = ( 20 ) 2 + ( 20 ) 2 = 20 2 m
12. (c)
13. (d) : Heat lost by metal ball = Heat gained by container and water mb sb (500 – ) = mc sc ( – 30) + mw sw( – 30) 0.1 × 400 (500 – )
0.5 × 4200( – 30) 40(500 – ) = 2900 ( – 30) 20000 – 40 = 2900 – 87000
107000
=
= 36.4°C
2940

Downloaded from jeemain.guru


Δθ 100 = 36 . 4 − 30
44% increase in the temperature of water = θ× 30
100 21 33
× = . %
14. (c) : Equivalent circuit is given as
50 150
I
6
6 V 100 I = 50 + 150

+ 100
= 300 6= 0 02. A
G G
15. (b) : E = 1000 V/m, p = 10–29 cm, = 45° Potential U = – p E⋅ energy = –pE stored cos
in the dipole,
1 10 − 26
= –10–29 × 1000 × cos45° = – 2 × = – 0.707 × 10–26 J –7 × 10–27 J
K ⎛⎜
16. (d) : In a hydrogen atom when an electron jumps from M – Shell to N-shell then, 1 λ = ⎝2

12 − 3
⎞⎟ = K ...(i) K ⎛⎜
12 ⎠ 36 5 for N to L shell, 1 λ ′ = ⎝2

12 − 4
⎞⎟ = K
12 ⎠ 16 3 = 16
3
×5
36λ = 20 27 λ
20λ
∴ λ ′= 27
17. (a) : I = I1 + I2 + I3 = I1 + 2 × I2 (' I2 = I3)
1 MR2 MR2
= 2 + 2 × (MR2 +
) = 3 M R2 l
4 18. (a) : Time period of a simple pendulum, T = 2 g
gp
GM
= R 2p
p

= 4 GMD 2 p
p

4
=( GMD 2 E
E

)
3
×3 2 gp
g
= e 3
'ge4
GME ⎛⎜
⎝= ⎞D E
2
T
⎟ ⎠∴ T

T = 2 3s
= 13 , p (Te = 2 s)
1240eV-nm
19. (c) : E1 =
= 4.13 eV 1240eV-nm
300nm E2 =
= 3.10 eV
400nm KKmaxmax1 1 = E1 – 0, Kmax2 = E2 – 0
magnetic moment
– Kmax2 = E1 – E2 = 1.03 eV V1 – V2 = 1.03 V (' K = eV) 20. (c) : Intensity of magnetisation I = volume
= 20 × 10
10 −6−

=
20

A/m

χ=H
I
= 60 20

× 10
3
V PV
= 13 × 10 − 3 = 3 . 3 × 10 − 4 21. (b) : VT = K , ⎛⎜ ⎝ R
⎞⎟ = K or PV2 = R +C = R
⎠ = constant For a polytropic process C 1− x V 1− 2
+ R=R 1 RT
32 2Q=nCT= 2 22. (c) : Potential difference between A and B = Potential difference between C and D = ( ε 1 / r 1
+
/ 2r 2)
+
r1
3

( ε 3/ r
3

1+2
)= 3
+3

= 63 = 2V

23. (b) : x0 Boiling water (100°C) x03 Ice (0°C) It means change in length on the linear scale, corresponds to 100°C.
2x 0 3 100
31 corresponds to ×

2x
⎛⎜ −
0 ⎝x 0 2 x30
⎞⎟ corresponds to 3 100

20
25
x6

C
R1
24. (c) : Case I : R2
×
x × 0 = ° = 4060
= 23 ...(i)
R 1 + 10 2 R 2 × 30 = 20
Case II : R2= 1 or, R1 + 10 = R2 or 3 2+ 10 = R2 ; R2 = 30 R1 = 3

Downloaded from jeemain.guru


45 Solved Paper 2019
Case III : Resistance R is connected in parallel with resistance (R1 + 10) ( )20 10
3030
=
1C3 4
+ =
1C C3 18
+
32
=
16
=
26 138 + + R
×
R

=
23

C3 = 13 8μF C4 is given by 3R = 60 + 2R 1 1R = 60 . 2
15
25. (a) : RG = 20 , Ig = 0.005 × 30 = 0.150 A To read 15 V (RG + R) Ig = 15 RG + R = 0
= 100 τ = 2 5 = = + =+=
15. R = 100 – RG = 100 – 20 = 80 . 26. (d) : As = r F sin sin = rF 5 × 1 5 14 52 11C 4 C C 4 C4 = 11 4μ
minimum deviation, i = e r1 = r2 = A2
°
= 602 = 30° Using snell’s law at the interface, 1 sin i = 3 sin r1

. 12

, θ = π 6 27. (a) : C1 is given by


1 13
sin i = 3 sin 30° = 3 2/ i = 60°. 29. (d) : Angular frequency of the pendulum.
= + + + + =
C1 C C1 C1 C1 C1 16
+ (' C = 2 F)
24
C1 = 13 6μF
geff Δ Δω= 1 1
= l, ω 2= 2
geff g
eff

C2 is given by
1 15
ω2
2A s g
ω = 12
× × −
× () 2 () 2= 10–3 rad/s
= Power
30. (d) : Intensity of electromagnetic wave is given by I
Area
2 1 10
10
= + = + =
C 2( C C2 10 1 1
11 P
10 (A ))
= Ec
12 0 C2 = 1011 μF C3 is given by
=2P
ε 2E Aε
0

= 2 × 27 ×
c
10

3

10 − 5 × 9 × 10 − 12 × 3 × 10 8 = 2× 103 V/m = 1.4 V/m


Downloaded from jeemain.guru
46 JEE Main

JEE MAIN
Held on Solved Pape
2019
12th January 1st SHIFT 1.
What is the position and nature of image
formed by lens combination shown in figure? (f1,f2 are focal lengths)
20 cm from point B at right, real
(a) 70 cm from point B at right; real (b) 3 (c) 40 cm from point B at right, real (d) 70 cm from point
2. A particle of mass m moves in a circular orbit
1
in a central potential field U ( r ) = 2
kr . (a) 25 (b) 22 (c) 5 (d) 12
4. In the figure shown, after the switch S is turned from position A to position B the energy dissipated
2 If
in the circuit in terms of capacitance C and total charge Q is Bohr’s quantization conditions are applied, radii of possible orbitals and e
with quan
5
(a) 8

(a) r n n2
∝ 1 r ∝n,E ∝ n ∝ , ∝ r ∝ n,E ∝ n1
,En n 2(b) n n (c) (d) r n n En n n n 3. The galvanometer deflection, when key K1 is
open, equals 0, (see figure). On closing K2 also and adjusting R2 to 5 , the deflection in galvanometer The resistance of the becomes g
θ
0then, 5 .
given by [Neglect the internal resistance of battery]
1
Q2C (b) 8
3
Q2C (c) 8

Q2C 5. There is a uniform spherically symmetric surface charge density at a distance R0 from the origin. The charge distribution is ini
starts expanding because of mutual repulsion. The figure that represents best the speed V(R(t)) of the distribution as a function of its ins
radius R(t) is
(a)
3
Q2C (d) 4
Downloaded from jeemain.guru
47 Solved Paper 2019
(b)
(c)
(d)
6. A point source of light, S is placed at a distance L in front of the centre of plane mirror of width d which is hanging vertically on a w
in front of the mirror along a line parallel to the mirror, at a distance 2L as shown below. The distance over which the man can see the
source in the mirror is
d
(a) 3d (b) 2d (c) d (d) 2 7. Two light identical springs of spring constant k are attached horizontally at the two ends of a uniform hori
length l and mass m. The rod is pivoted at its centre O and can rotate freely in horizontal plane. The other ends of the two springs are fi
supports as shown in figure. The rod is gently pushed through a small angle and released. The frequency of resulting oscillation is
1
(a) 2π
1
km (b) 2 π

6 km
1
(c) 2 π
1
3 km (d) 2 π
2 km
8. As shown in the figure, two infinitely long, identical wires are bent by 90° and placed in such a way that the segments LP and QM a
axis, while segments PS and QN are parallel to the y-axis. If OP = OQ = 4 cm, and the magnitude of the magnetic field at O is 10–4 T
wires carry equal currents (see figure), the magnitude of the current in each wire and the direction of the magnetic field at O will be ( 0
(a) 20 A, perpendicular into the page (b) 40 A, perpendicular into the page (c) 20 A, perpendicular out of the page (d) 40 A, perpendicu
page. 9. A passenger train of length 60 m travels at a speed of 80 km/hr. Another freight train of length 120 m travels at a speed of 30 k
of times taken by the passenger train to completely cross the freight train when: (i) they are moving in the same direction, and (ii) in the
25(b) 3 (c) 5 (d) 11 G
direction is (a) 11 2 2 5 10. The position vector of mass the r cm centre of of an asymmetric uniform bar of negli
cross-section as shown in figure is
L
L 2L
3L

Downloaded from jeemain.guru


48
G 11
(a) r cm = 8

Lx ^ + 38
Ly ^
G 13
(b) r cm = 8

Lx ^ + 58
Ly ^
2 K + 3K
(a) 1 25

K +
(b) 1
K K + 3K
2 2(c) K1 + K2 (d) 1 4

Lx ^ + 118
Ly ^
G 5
(d) r cm = 8

Lx ^ + 138
Ly ^
11. An ideal battery of 4 V and resistance R are connected in series in the primary circuit of a potentiometer of length 1 m and resistanc
R, to give a potential difference of
15. A light wave is incident normally on a glass slab of refractive index 1.5. If 4% of light gets reflected and the amplitude of the electr
incident light is 30 V m–1, then the amplitude of the electric field for the wave propagating in the glass medium will be (a) 24 V m –1 (b)
V m–1 (d) 6 V m–1
5 mV across 10 cm of potentiometer wire is
16. A travelling harmonic wave is represented (a) 490 (b) 495
by the equation y(x, t) = 10–3 sin(50t + 2x), (c) 395 (d) 480
where x and y are in meter and t is in seconds. 12. Determine the electric dipole moment of
Which of the following is a correct statement about the wave? the system of three charges, placed on
(a) The wave is propagating along the
positive x-axis with speed 25 m s–1. the vertices of an
(b) The wave is propagating along the equilateral triangle, as
positive x-axis with speed 100 m s–1. shown in the figure.
(c) The wave is propagating along the
^
(a) 2ql j(b) ( )
^j i i+ j
negative x-axis with speed 25 m s–1. (d) The wave is propagating along the (c) 3 ql − ^ 2 ql ^ ^ 2 (d) − 3ql ^
–1
jnegative x-axis with speed 100 m s .
17. A simple pendulum, made of a string of
13. A satellite of mass M is in a circular orbit of radius R about the centre of the earth. A meteorite of the same mass, falling towards
with the satellite completely inelastically. The speeds of the satellite
length l and a bob of mass m, is released from a small angle 0. It strikes a block of mass M, kept on a horizontal surface at

oscillations, elastically. It bounces back and goes up to an angle 1. Then M is given by and meteorite are the same, just before
subsequ
m
(a) 2 combined body will be (a) in an elliptical orbit (b) in the same circular orbit of radius R (c) in a circular orbit of a different radi
escapes to infinity. 14. A cylinder of radius R is surrounded by a cylindrical shell of inner radius R and outer radius 2R. The thermal c
material of the inner cylinder is K1 and that of the outer cylinder is K2. Assuming no loss of heat, the effective thermal conductivity of t
heat flowing along the length of the cylinder is
(b) m
⎛⎜ ⎝⎞⎟ ⎠ ⎛⎜
θ θ
⎝ θ θ
m
⎞⎟ ⎠(c) 2

θ0 θ
1
0 +1 θ 0 +
θ1
0−
(d) m
1 ⎛⎜
⎝⎞⎟ ⎠ ⎛⎜
θ θ
⎝ θ θ
⎞⎟ ⎠18. For the given
cyclic process CAB as shown for a gas, the work done is (a) 10 J (b) 1 J (c) 5 J (d) 30 J
+
θ0 θ
1
0 −1 θ 0 −
θ1
0+
1

Downloaded from jeemain.guru


49 Solved Paper 2019
dR of its resistance R with
19. In a meter bridge, the wire of length 1 m has a non-uniform cross-section such that the variation dl leng
l . Two equal resistances are connected as shown in the figure. The galvanometer has zero deflection when the jockey is at point P. Wha
?
(1– )l (a) 0.35 m (b) 0.2 m (c) 0.25 m (d) 0.3 m
20. The least count of the main scale of a screw gauge is 1 mm. The minimum number of divisions on its circular scale required to mea
diameter of a wire is (a) 50 (b) 100 (c) 200 (d) 500
21. A person standing on an open ground hears the sound of a jet aeroplane, coming from north at an angle 60° with ground level. But
3v 2v v
aeroplane right vertically above his position. If v is the speed of sound, speed of the plane is (a) v (b) 2 (c) 3(d) 2 22. Two elec
at (25 W, 220 V) and (100 W, 220 V), are connected in series across a 220 V voltage source. If the 25 W and 100 W bulbs draw power
respectively, then (a) P1 = 9 W, P2 = 16 W (b) P1 = 16 W, P2 = 4 W (c) P1 = 4 W, P2 = 16 W (d) P1 = 16 W, P2 = 9 W
23. The output of the given logic circuit is
(a) AB (b) AB (c) AB + AB (d) AB + AB 24. A proton and an -particle (with their masses in the ratio 1 : 4 and charges in the ratio of
accelerated from rest through a potential difference V. If a uniform magnetic field (B) is set up perpendicular to their velocities, the rati
r of the circular paths described by them will be (a) 1 : 3 (b) 1 2: (c) 1 : 2 (d) 1 3:
25. An ideal gas occupies a volume of 2 m3 at a pressure of 3 × 106 Pa. The energy of the gas is (a) 9 × 106 J (b) 3 × 102 J (c) 6 × 104 J
26. A 100 V carrier wave is made to vary between 160 V and 40 V by a modulating signal. What is the modulation index? (a) 0.4 (b) 0
27. A straight rod of length L extends from x = a to x = L + a. The gravitational force it exerts on a point mass m at x = 0, if the mass
the rod is A + Bx2, is given by
⎡ ⎢ 1
(a) Gm ⎣ A ⎛⎜ ⎝ a − a +
1 ⎞⎟ − ⎤ ⎥ ⎡ ⎢ ⎛⎜ 1 − 1 ⎞⎟ − ⎤ ⎥ ⎡ ⎢ ⎛⎜ 1 − 1 ⎞⎟
L ⎠ BL ⎦ (b) Gm ⎣ A ⎝a + L a ⎠ BL ⎦ (c) Gm ⎣A ⎝a + L a ⎠
⎡ ⎢ 1 1 ⎞⎟
Gm ⎣ A ⎛⎜ ⎝a −a+ L
Downloaded from jeemain.guru
5028. In the figure shown, a circuit contains two identical resistors with resistance R = 5 and an inductance with L = 2 mH. An ideal
is connected in the circuit. What will be the current through the battery long after the switch is closed ?
(a) 3 A (b) 5.5 A (c) 7.5 A (d) 6 A
29. Let the moment of inertia of a hollow cylinder of length 30 cm (inner radius 10 cm and outer radius 20 cm), about its axis be I. The
cylinder of the same mass such that its moment of inertia about its axis is also I, is (a) 14 cm (b) 16 cm (d) 12 cm (d) 18 cm 30. A parti
and charge q is accelerated by a potential difference of 50 V. Another particle B of mass 4 m and charge q is accelerated by a potential

2500 V. The ratio of de-Broglie wavelengths λAλB is close to (a) 14.14 (b) 0.07 (c) 4.47 (d) 10.00 SOLUTIONS
1v −u1 =
1. (a) : For the first lens, 1 1

1f 1
1⇒ + 20
1 GI
= ⇒ = = 5I3 = GI4 I3 = 45

⎛⎜ G ⎞⎟ =
Also, I3 + I4 = I2 ⇒ ⎝ 5+1 ⎠I 4 I2
⇒=
++
v1
RI2 R1 2 I3v 1 15 Now, for the secon
= . cm
u2 667
− = 14 cm
2 3 1 1 1 1 1I4
V

5
1
G
− = ⇒ = +
v2 u2 f2 v2 − 14 3v2 = 70 cm right of second lens.

2. (b) : Force F = ∂
∂ =−∂
r due to this field F ∂
U 4
r kr kr I
R 1G
V5

G
G5
5 5 For a galvan
⎞⎟ +⎛⎜ ⎞⎟ = θ
⎛⎜ ⎝ ⎠ ⎝ ⎠ θ0
V
+
G
R 1( G + 5 )
+5
1
G ⎛⎜ ⎝ 2
⎞⎟ = − mv
2 ⎠ For circular orbit, 2

kr v r
r =− ⇒ ∝
...(i)
5
V 25 (220 + G) = (220) (G + 5) + 5G 25 (220) + 20G = 220 G + 1100 200G = 4400 G = 22 Also, by Bohr’s quantization condition m
eqn. (
4. (c) : Initially, the energy stored in the circuit ...(ii)
Q2
is
.
2 C When the switch S is turned into position B, the U ( r )= net capacitance becomes C + 3C = 4C and total
Q
charge Q remains the same. En So, the energy stored will be C
1
2
1
n kr 2 E n =− 2

U(r)
= V
=− 14 kr 2 3. (b) : Initially, let I1 be the current through G. then I 1 220
.
+ G After Apply the KVL key on Kloop 2 is closed, 1,
the circuit is shown as
= .
222 (4) 8 So, the difference of energy is dissipated in the given s

QC
8 2− 2
2= − 38 2
.
5. (d)

QQC C
Downloaded from jeemain.guru
51 Solved Paper 2019
6. (a) : In the given figure AED and ABC are
So, ED
BC
BE

t1 = A

= AD

AC⇒ ED
2
BC=

L Ld DC S 50 ⇒ BC = 2 ED ...(i)
Also, AED and ASD are
180
×
3600
10
3

×
= 18 25 18
s
When the trains are moving in opposite direction relative So time |v1 – velocity,
taken (–v2)| cross = 80 each + 30 other
= 110 km hr–1
congruent triangles. So, ED = DS Using (i) and (ii), BC = d
180
t2 = 110 ...(ii)
1000
3600 So, the distance over which the man can see the image of the light sour

d + d + d = 3d 7. (b) : Let the rod be rotated through a small angle . Due to restoring force of the spring, the torque acting on the rod i
⎞⎟ + l ⎛⎜ l ⎞⎟ ⎛⎜ l ⎞⎟ + l ⎛⎜ l ⎞⎟ = l 2
⎠( kx ) 2 ( kx ) = ⎝2 ⎠ ⎝k 2 θ ⎠ 2 (k) ⎝2 θ ⎠

2...(i) A
...(ii)

Using equations (i) and (ii), α ml l k θ α m k

θ L 2L
18 ×
×
= 36
110 s
t
Ratio 1t2
18 ×18
=

18 25 × 36
= 115 110
10. (b) : r 1 = L ( x + y )
L =
r 2 = 2 Lx + 2 y r r G 3 cm = 2.
mr G + mr G +
5 Lx m
mr G
+m+
my
→ r1r→2 L → 11 22 33 L 2L r3
x
3L
r ⎡ ⎢ = ⇒ = 6
1 2 3 Let ∴ m cm 2 = = m, so ⎣ m3 = m and m1 = 2m 2 2 12 2 6 α= m
k L x y ⎛⎜ Lx L ⎞⎟ +
θ = ω 2 θ 14 2 ( + )+ ⎝2 + 2y ⎠ 2.
⎤ ⎥ 1 65 25 13 ω
5 Lx ⎦ = 4 + = 8 + ⇒ω = = π=π
L L ;
(.x . y) x 5 8 y.6mk
T2
k
16 2 m 8. (a) : The magnetic field due to wires LP and MQ will be zero at point O. Magnetic field at point O due to the vertical wir
μ
B1+B2= 4π
0r
I 0 90
(sin + sin ° ) + μ4 π 0

0 90 4 4π
r I(sin + sin ° ) 10 − = 2 ×
×

10

7 20
× 0 . I 04 ⇒ I = A

11. (c) : Let I be the current in the circuit. 4 = (5 + R) I ...(i) According to given condition,
⎛⎜ 10
5 × 10 − 3 = ⎝100
⎞⎟ 5
⎠( )( )I I = 10–2 A ...(ii) Using (i) and (ii), 5 + R = 400

R = 395 12. (d) : The given system of charges can be considered to consists of two dipoles of charges +q and –q. Let p be the dipo
dipole of charges +q and –q (placed at point B). By right hand thumb rule direction of magnetic field at O will be perpendicular into
The total distance to b
30°
train is 60 + 120 = 180 m. When the trains are moving in the same direction, relative time taken velocity to cross is v1 each – v2 other.
= 80 – 30 = 50 km hr–1 so
y
p sin 30° B
–2q p sin 30°

ppA GC
+q 2 p cos 30° The horizontal components of +qp x

(due to charges
Downloaded from jeemain.guru
p cos °
52at A, B and B) cancels each other and vertical components adds up. So, the net dipole moment of system of charges, 2 30
ql ) ⎛⎜ 3⎞⎟ ( − j ) = − 3 ql j
⎝2 ⎠ . 13. (c) : Due to inelastic collision, the total energy of the systems changes.
It will shift in a circular orbit of different radius.
14. (d) : We know that thermal resistance R = For inner For outer R 2
=l
cylinder cylinder K 2π
3 R 2R 1R 2
==l
K 1π
l
R 2 K 2 π[( 2 R ) 2 − ( R ) 2 ]
l
KA
Req is the equivalent thermal resistance of the cylinder
R =R +
1 eq 11
R 4
1 2 4 K Keqeq l=π
3 =K +
π 2+ 2π
2
K eq 1 4
3K
2 15. (a) : Let components of Eot the and electric Eoi be the perpendicular field for transmitted and incident waves respectively. Eo
2
=1 +KRl
μ
1μ 1μ
2
1
( )15 .
×4 = 24 V m–1
= 45 Eot = 30 5 16. (c) : The given
wave is y(x, t) = 10–3 sin (50t + 2x) Comparing with standard equation
ω
y (x, t) = a sin ( t + x) Velocity v = β

= 502 = 25 m s − 1 In the given equation x is positive. Therefore, the wave is travelling in the negative x-direction. 17. (d) :
M
θ0 l
θ1
l
u
m
v
m
Just before collision
Apply conservation of energy,
= m −M
u = 2 gl ( 1 − cos θ 0 ) ...(i) v = velocity of bob after collision v ⎛⎜ ⎝
m+
⎞⎟ u Bob rises up to angle = m −M
M ⎠ 1v = 2 gl ( 1 − cos θ 1 ) v ⎛⎜ ⎝
m+
⎞⎟ u = 2 gl ( 1 − cos θ ) ...(ii)
M ⎠ 1 from eq. (i) and (ii)

m m + −M M
1−
= cos
1
θ
−cos 1θ 0

{ cos θ =
1

2 sin
2

θ
} 2 m m + −M
M
=
sin sin
⎞⎟ ⎞⎟
⎛⎜ ⎝⎛⎜ ⎝θθ22 10 ⎠ ⎠( small) Mm = θ θ 0 0 +

θθ
1
1

, M = ⎛⎜
θ
1
⎞⎟
⎠m 18. (a) : CA = change in volume AB = change in pressure P = (6 V – = 1) (5 = – 5 1) m= 3
4 m3
1 4 5 10 J
Work done = Area of ABC W = 2 × × =
1 × AB × CA
= 2
19. (c) : Let AP be the length l
1 = dl
dRdl ∝ l , dR K l Taking integration on both the sides

∫ dR = K ∫ 1 l dl R = 2Kl1/2 = 2K ( l = 1 m) Balancing point will divide the resistance in equal part. So, l will be correspond to K ( ).
L.C. 1
K = 2 K l or, l = 0.25 m 20. (c) : Least count = 1 mm Nd (Number of divisions) = m5 μ = 5

mm mμ
= 105 3 = 200
=
21. (d) : v p vcos60
°

vp

v2
60°
30° v

Downloaded from jeemain.guru


53 Solved Paper 2019
22. (b) :
Resistance across 25 W bulb
R1

V = (μ )
P P= 25 W V = 220 V P= 100 W V = 220 V 27. (d) : dF Gm x 2 dx F=G

) A+
(
Bx
x 220 V

2
2
dx
)
==
= 2⎛x F = Gm ⎜⎜
+
⎝A x a
L ∫ x−
2
dx + =a
= +
x a

L ∫ B⋅
dx 1
= ×
220 25 220
1936
= Ω
Resistance across 100 W bulb
R2
⎛⎜
⎞x =a F = Gm
−⎡ ⎢ =V
⎝A ⎣ x ⎟⎟ ⎠ P2
= ×
2 220 100 220
484 ⎤ ⎥ = V = 220
= Ω ⎦ a aTotal Resistance Total current I R R 1 + R2420 2
= 2420
= 0 09. A
Power I1 + L
+
B[x
⎛⎜ ⎡ ⎢ 1 1⎤ ⎥ + + − ⎞⎟
] a a+ L ⎞⎟ ⎠F = Gm ⎝− A ⎣ a+ L−a ⎦ Ba [ L a] ⎠F = Gm A a − a + L BL Power

consumed 1 2 = = 0.09 0.09


0.09 × 1936 16 W by 100 W bulb 0.09 × 4
y
⎛⎜ 11⎤ ⎥ + ⎞⎟
⎝⎡ ⎢ ⎣ ⎦ ⎠28. (d) : The coil offers zero resistance, after the switch is closed for a long tim
·( AB )
V = 15 V R1 R
_
B A·BB
B
(A_

= V
·B+B)I ( R × R )/(
=V
R+R) R
×R ×
= Y = A ⋅ ( AB ) ⋅ ( AB + B )
(by applying De Morgan theorem) Y = A ⋅ ( A + B ) + ( AB ⋅ B ) Y = A ⋅ ( A + B ) + ( AB ⋅ B ) ( ' B B⋅ = 0 ) Y = A ⋅ A +
(b) : We know that
mv
r Bq
2 V2 R
2
=6A

m(R 2 +R 2)
29. (b) : 1 2 2 =
mK 2
Km 2 ( = 10 250
2 +
2 20
2 100 +
) = mK 2 2
400

= K 2 K 16 cm 30. (a) : For particle A


=B
12 ∝m
mVq r q Given m
m
m1 = m, q1 = q; V1 = 50 V
=1
p λA = h2 mqV 1 1 1 α 4
;
qp q
α ...(i)
=
12 For particle B rp r α
= m pq
αq p m
⎛⎜ 1
α= ⎝ 4

m2 = 4m, q2 = q; V2 = 2500 V λB = h2 mqV 2 2 2


⎞⎟ =
⎠ 12
3
25. (a) : We know that E =
PV 3 3 × 10 2 E
2= × 2 PV = 3
2
×
...(ii)
A −A
Dividing eqn. (i) by (ii), 6 = 9 × 106 J λA26. (c) : Modulation Index λB μ = A +A
=− ===
+ mqV mqV
= ×
4m q
×
2500 m × q ×
50 λAλB 2 2 2 1 1 1
4× 2500
max min max min = 50
14 14. 160 40
= 160 40
0.6
120 200

Downloaded from jeemain.guru

JEE MAIN
54 JEE Main Held on Sol
Paper 2
12th January 2nd SHIFT 1.
A block kept on a rough inclined plane, as
directly backwards losing, 64% of its initial shown in the figure, remains at rest upto a
kinetic energy. The mass of the nucleus is maximum force 2 N down the inclined plane.
(a) 4 m (b) 1.5 m The maximum external force up the inclined
(c) 3.5 m (d) 2 m plane that does not move the block is 10 N. The coefficient of static friction between the block and the plane is [Take
4. A simple harmonic motion is represented by
y = 5 (sin 3 π t + 3 cos 3 π t ) cm The amplitude and time period of the motion
cm s, 3 (b) 10 cm s, 2 cm s, 2 (d) 10 cm s, 3 1 (b)
are (a) 5 2 3 (c) 5 3 2 5. A long cylindrical vessel is half filled with a (a) 2
liquid. When the vessel is rotated about its own vertical axis, the liquid rises up near
2. Formation of real image using a biconvex lens
is shown below
the wall. If the radius of vessel is 5 cm and its rotational speed is 2 rotations per second, then the difference in the heights between the c
sides, in cm, will be (a) 0.4 (b) 2.0 (c) 0.1 (d) 1.2
6. A 10 m long horizontal wire extends from
If the whole set up is immersed in water without disturbing the object and the screen positions, what will one observe on the screen?
North East to South West. It is falling with a speed of 5.0 m s –1, at right angles to the horizontal component of the earth’s magnetic f
Wb/m2. The value of the induced emf in wire is (a)
(a) 0.3 × 10–3 V (b) 2.5 × 10–3 V (b) Magnified image
(c) 1.5 × 10–3 V (d) 1.1 × 10–3 V (c) Erect real image (d) No change
7. A soap bubble, blown by a mechanical pump at the mouth of a tube, increases in volume, 3. An alpha-particle
with time, at a constant rate. The graph that 1-dimensional elastic collision with a nucleus
correctly depicts the time dependence of at rest of unknown mass. It is scattered
pressure inside the bubble is given by
Downloaded from jeemain.guru
55 Solved Paper 2019
(a) (b)
(c) (d)
8.
3 μF, R = 20 ,
In the given and R1 = circuit, 10 . Current C = in 2 L – R 12 path is I1 and in C – R2 path it is I2. The voltage of A.C s
V = 200 2 100sin( t ) volts. The phase difference between I1 and I2 is (a) 0 (b) 30° (c) 90° (d) 60° 9. An ideal gas is enclosed in a cylin
2 atm and temperature, 300 K. The mean time between two successive collisions is 6 × 10 –8 s. If the pressure is doubled and temperatu
500 K, the mean time between two successive collisions will be close to (a) 4 × 10–8 s (b) 3 × 10–6 s (c) 0.5 × 10–8 s (d) 2 × 10–7 s 10.
experiment, an electron of energy 5.6 eV passes through mercury vapour and emerges with an energy 0.7 eV. The minimum wavelen
emitted by mercury atoms is close to (a) 220 nm (b) 1700 nm (c) 250 nm (d) 2020 nm 11. A vertical closed cylinder is separated into
frictionless piston of mass m and negligible thickness. The piston is free to move along the length of the cylinder. The length of the cy
piston
is l1 and that below the piston is l2, such that l1 > l2. Each part of the cylinder contains n moles of an ideal gas at equal temperature T.
RT
stationary, its mass m, will be given by (R is universal gas constant and g is the acceleration due to gravity) (a) ng
⎡ ⎢ +
⎣ 12 1
3−

⎤ ⎥ (b) nRT
⎦ gll
⎡ ⎢ +
⎣ 211 22
⎤ ⎥ (d) nRT
⎦ g
⎡ ⎢ −
⎣ 11 22

⎤ ⎥
⎦ 12. A load of mass M kg is suspended from a steel wire of length 2 m and radius 1.0 mm in Searle’s apparatus experiment. The
length produced in the wire is 4.0 mm. Now the load is fully immersed in a liquid of relative density 2. The relative density of the mate
The new value of increase in length of the steel wire is (a) 4.0 mm (b) zero (c) 5.0 mm (d) 3.0 mm
13. A parallel plate capacitor with plates of area 1 m2 each, are at a separation of 0.1 m. If the electric field between the plates is 100 N C
of charge on each plate is
⎛⎜ ε = 8 85 × 10 12 C2
⎝l l l l Take 0 Nm 2 l l l l

. − ⎞⎟ ⎠(a) 8.85 × 10–10 C (b) 7.85 × 10–10 C (c) 9.85 × 10–10 C (d) 6.85 × 10–10 C
14. The charge on a capacitor plate in a circuit, as a function of time, is shown in the figure.
What is the value of current at t = 4 s? (a) 3 A (b) zero (c) 1.5 A (d) 2 A
15. In the circuit shown, find C if the effective capacitance of the whole circuit is to be 0.5
F. All values in the circuit are in F.
Downloaded from jeemain.guru 6

5
6 μF (b) 4 F 7 μF (d) 7 μF , RC = 1 k and VBE = 1.0 V. The minimum base
(a) 5 (c) 11 10 16.
current and the input voltage at which the
Let l, r, c and v represent inductance, resistance,
transistor will go to saturation, will be
capacitance and voltage, respectively. The
respectively (a) 20 A and 3.5 V (b) 25 A and 3.5
l in SI units will be
dimension of rcv (a) [LA–2] V (c) 20 A and 2.8 V (d) 25 A and 2.8 V 21. In
(b) [LT2] (c) [A–1] (d) [LTA] 17. Two satellites, the given circuit diagram, the currents, I1 = 0.3
A and B, have masses m and 2 m respectively. A A, I4 = 0.8 A and I5 = 0.4 A, are flowing as
is in a circular orbit of radius R, and B is in a shown. The currents I2, I3 and I6, respectively,
circular orbit of radius 2R around the earth. The
are
T A , is
ratio of their kinetic energies, T B (a) 1 (b)
1 (d) 1
2 (c) 2 2 18. The mean intensity of
radiation on the surface of the Sun is about 108
W/m2. The rms value of the corresponding
magnetic field is closest to(a) 10–2 T (b) 1 T (c) (a) 1.1 A, 0.4 A, 0.4 A (b) –0.4 A, 0.4 A, 1.1 A
–4 2 (c) 1.1 A, –0.4 A, 0.4 A (d) 0.4 A, 1.1 A, 0.4 A
10 T (d) 10 T 19. A resonance tube is old and
has jagged end. It is still used in the laboratory to 22. When a certain photosensitive surface is
determine velocity of sound in air. A tuning fork illuminated with monochromatic light of
of frequency 512 Hz produces first resonance frequency , the stopping potential for the photo
when the tube is filled with water to a mark 11 −V
current is 02 .When the surface is illuminated
cm below a reference mark, near the open end of
the tube. The experiment is repeated with
υ , the
by monochromatic light of frequency 2
another fork of frequency 256 Hz which stopping potential is –V .
produces first resonance when water reaches a 0 The threshold
mark 27 cm below the reference mark. The 4 υ
frequency for photoelectric emission is (a) 3
velocity of sound in air, obtained in the
experiment, is close to (a) 335 m s–1 (b) 322 m s– 5 υ 3 υ
(b) 2 (c) 3 (d) 2 23. A paramagnetic
1
(c) 328 m s–1 (d) 341 m s–1 material has 1028 atoms/m3. Its magnetic
20
susceptibility at temperature 350 K is 2.8 × 10–4.
.
Its susceptibility at 300 K is (a) 3.267 × 10–4 (b)
3.672 × 10–4 (c) 2.672 × 10–4 (d) 3.726 × 10–4

24. A plano-convex lens (focal length f2,


refractive index 2, radius of curvature R) fits

In the figure, given that VBB supply can vary exactly into a plano-concave lens (focal length f1,
from 0 to 5.0 V, VCC = 5 V, dc = 200, RB = 100 k
Downloaded from jeemain.guru
57 Solved Paper 2019
refractive index 1, radius of curvature R). Their plane surfaces are parallel to each other. Then, the focal length of the combination wil
R
μ 2 − μ 1 (c) f1 – f2 (d) 2 1 2 1 2
(b)
ff+
f f (c) 25. A galvanometer, whose resistance is 50 ohm, has 25 divisions in it. When a current of 4 × 10 –4 A passes through it, it
deflects by one division. To use this galvanometer as a voltmeter
(d)
should be connected to a resistance of (a) 200 ohm (b) 6250 ohm (c) 6200 ohm (d) 250 ohm
28. To double the covering range of a TV 26. Two particles A, B are moving on two
transmission tower, its height should be concentric circles of radii R1 and R2 with equal
multiplied by angular speed . At t = 0, their positions and direction of motion are shown in the figure.
1 (b) 2 (c) 4 (d) 2
(a) 2 29. In a radioactive decay chain, the initial nucleus is 23290 Th. At the end there are 6 -particles and 4 -par

emitted. If the end nucleus is ZAX, A and Z are given by (a) A = 202; Z = 80 (b) A = 200; Z = 81 (c) A = 208; Z = 80 (d) A = 2
G G πis
The relative velocity given by (a) − +ω( R 1 R 2 ) i ˆ (c) ω( R 2 − R 1 ) i v A−v B at t = ω2
ˆ
(b) (d) ω( − +ω( R 1 R 1 − R 2 R ) 2 i
)iˆ
30. A particle of mass 20 g is released with an initial velocity 5 m/s along the curve from the point A, as shown in the figure. The poi
from point B. The particle slides along the frictionless surface. When 27. The moment of inertia of a solid sphere, about an axis paral

the particle reaches point B, its angular momentum about O will be (Take g = 10 m/s2) distance of x from it, is I(x). Which one of the g
the variation of I(x) with x correctly?
(a)
(a) 6 kg-m2/s (b) 8 kg-m2s (c) 3 kg-m2s (d) 2 kg-m2/s
Downloaded from jeemain.guru
58

SOLUTIONS
1. (b) : Case I. When 2 N force is acting on the block resolve the forces along and perpendicular the plane.
N
2
g sin
m mg
mg cos
Substituting (ii) and (iii) in eqn. (i) N f
=−⎛⎜
mv ⎝10
6 ⎞⎟ + ⎛⎜
v ⎠m M ⎝10
m⎞⎟ ⇒
8v M ⎠ 1610
mv = v mM 2 5 ⎛⎜
10 8 M = 4m 4. (b) : The given equation is y = 5 (sin 3 π t + 3 cos 3 π t ) – 2 = mg sin – mg cos ...(i) y = × ⎝C
force 10 N is acting on the block, free body diagram of the block i
N
0N
1
1 sin 3 π 3cos 3 π ⎞⎟ 10 ⎛⎜ cos π sin 3 sin π cos 3 ⎞⎟ 10 sin ⎛⎜ 3 π π ⎞⎟
2 t+2 t ⎠= ⎝ 3 πt+ 3 πt ⎠y = ⎝ t+ 3 ⎠...(i) C
with standard equation
y = A sin ( t + )
g sin f
= 3 and A = 10 cm m
mg
mg cos 2 2
Time period T = = 3
mg sin + mg cos = 10 ...(ii) Dividing eq. (i) by eq. (ii),
gggg
πω s 5. (b) : The linear speed of the liquid at the sides is r . So, the difference in height is given as
θ −μ
sin sin θ + μ cos
θ
cos θ
=
10
− 2; −
1 1 + μμ 3 3
=−
5
1⇒ μ = 32 follows 2gh = 2. (a) : The focal length of the lens is given

= ⎛⎜ μ − ⎞⎟ ⎛ ⎜ − ⎞⎟
1f ⎝ 2μ 1 1 ⎠ 1 ⎝R 1 R12 ⎠If the setup is immersed in water, the focal length of the lens increases. Hence
further away from the lens; while the position of screen has been kept unchanged. So, the image will not be observed on the screen.
1
3. (a) : Let M be the mass of the nucleus. Applying conservation of linear momentum, mv = mv1 + Mv2 ...(i) Also, 2

− 6. ( 2 (c) : The 2 π ) 2 ( 5 10
0 02
2 ) 2 2 10 .

G G
m motional emf is given as ε = v ( l×B ) = 5 × (0.3× 10–4) (10) sin90° = 1.5 × 10–3 V
dV
7. (*) : dt
const. or,
⎛⎜ ⎝43

π
⎞⎟ dr =K r dr = K 3 ∝1
3 ⎠= 43 π ( 3 r 2 ) dt , 4 π 2 dt r = Kt + C r = K1t1/3 + C1 The excess pressure inside the i.e. P excess ra
bubble is P
=P0
S
+ K 1t 13

+C1
=
dr d
36
12
mv 2 ⇒ v = v ...(i)
1 10 6 Applying conservation of kinetic energy, 1 2
4/
mv 2 = 12

mv 12 +
12

Mv 22 *None of the given options is correct.


8. (*) : For current I1
12
Mv 2 =
2 100 64

12
mv 2 ⇒ v =
2

vm X
10 8 M tanφ = R

100 10 ×
10
3 3
=

Downloaded from jeemain.guru


59 Solved Paper 2019
= 60° ; V leads I1. For current I2, tan = = =
××−
× = (100) (1) (8.85 × 10–12)
ω
q = 8.85 × 10–10C XC1 R 2 CR 2 100 90° ; V lags I2.
310
12 6
20 = 1000
3
The required phases difference between I1 and I2 is + = 60° + 90° = 150°
14. (b) : As the charge is constant on the plate for t = 2 s to t = 6 s. So for t = 4s, the current will be zero 15. (c) : The given circuit ca
follows: C*None of the given options is correct
A
9. (a) : Mean time between two successive
4/3 1
AB
collisions τ ∝ ×
C
BSo,

C
C 7/3
1 velocity number of particles
per unit volume
1
τ∝
= ∴ τ 1 = τ 2 21
7PT73

3
⎞⎟
⎛⎜ ⎝ ⎠P

T
+
510

711
τ2

TP
T1 = ⇒ C
= μF T
= ×
P2 6 102
− 8 500
15 10 4
300 = × −8 ×
10
−8s

10. (c) : The minimum wavelength of emitted photons is EΔ hc= −


⎡⎢ l⎤ ⎥ = [ ]T
16. (c) : For LR circuit, time = ⎣r ⎦ For capacitor C, [q] = [cv] = [AT]
250 nm
1240 56 07. .

So, ⎡ ⎢ ⎣ rcv
l⎤ ⎥ = [T]
⎦ [AT]
= A[ − 1 ] GM .
17. (a) : The velocity of the satellite is r 11. (d) : Let A be area of cross section of the cylinder. As the piston is at rest,
from both
TT

the sides should be equal mgA

1AB = 212
m A m B v v 22 B A

= 12

×2R
R=

1
+ nRTV
1=
nRTV 2
B μ rms 2
18. (c) : Mean intensity = 0

=
8 − Brms 10 10–4 T
×

×

10 3 × 10 19. (c) : Due to jagged end 4 ( 11 4 ( 27 − − x x vv) ) × × 10 10 − − 2 12 = = 5


=
1 nRT Al 2
⇒ m = nRT
g
−⎛⎜
⎝l 1 l 1 l 2 l
⎞⎟
2 ⎠12. (d) : Let and be the density of the liquid and material of the load respectively. In first case, the extension in the wire is
...(i) x = Mg/k = V g/k ...(i) When the load is immersed in the liquid,
...(ii) upthrust + internal force due to extension wire = weight of the load
= Vgx
V g + kx1 = V g x 1 = Vg ( ρ − σ )/ k Using (i) and (ii), x 1 Vg
( ρ − σ ) = x ⎛⎜ 1 − σ
ρ ⎝ ρ in

From eqn. (i) and (ii) 2(11 – x) = (27 – x) x = – 5 cm From eqn. (i) 4 × 16 v
512 ⎞⎟ = 4 × ⎛⎜ 1 − 2 ⎞⎟
× 10 − 2 = v 328 m s–1 20. (b) : Applying KVL at output and input circuit VCB = VCC – ICRC ⎠ ⎝ 8 ⎠
= σ =
The electric field between two plates is E ε 0 A
0

Downloaded from jeemain.guru


60VBB = IBRB + VBE ...(i) At saturation, VCB = 0 VCC = ICRC
=V
IC R
CC = 5
C 1 kv
= 5 mA I
Ω Base current, I B =
C 5 25 A
β = 200 = μ Using equation (i), VBB = (25 × 10–6) (100 × 103) + 1 = 2.5 + 1 = 3.5 V 21. (a) : Applying KCL at junction P1
I6 = 0.4 A From KCL at point R, I1 + I2 = I4
I2 = I4 – I1 = 0.8 – (–0.3) = 1.1 A At point S, I5 + I3 = I4 I3 = I4 – I5
I3 = 0.8 – 0.4 = 0.4 A. 22. (d) : Einstein’s photoelectric equation in the two cases is given by eV 2 0 = h −υ h υ 0 From eqn. (i) and (ii
− h υ
h υ eV υ / 2 0 − h = h υ υ 02 0 − h υ 0 ...(ii) 23. (a) : For a paramagne
υ 13 = T =
χ∝T 2χ2 χ 1T 12 28
= 3.267 × 10–4
300
f = − R
24. (b) : The focal length of the two lens is given by1 2 (μ2 1 ) ⎛⎜ ⎝ 1 2


⎞⎟ = μ
1 ⎠
R − f = − ⎛
2 2 11 1 (μ1 1)
⎞⎟ = − μ

R−
1 11 So, the focal length of the le
ffff

⎟ −
1 ⎠⎛⎜ ⎝
R
1
1

− 2+ 1

= −
μ
μ
μ2
R2

−1 μ
1

1
=μ2R

−μ 1
25. (a) : The full scale deflection current
= V
Ig = 25 × 4 × 10–4A Let R be the resistance connected I g R+
R g11
⇒ × × = .
25 4 10 − 4 2
5R +
π is
50 R = 200 26. (c) : The angle transversed in time ω2
ωπ
=t= ω
π=
π i.e., at shown t in = 2 the ω2
figure
, the position of two particles is
2
Y
AB
G G
The relative velocity v A−v B is
ˆ
=− = −ω( R R 1 ω 2 i − R ( 1 ) − i ˆ
ˆ
R 2ω i )

27. (b) : The moment of inertia I(x) at distance x is I ( x ) = 25

MR + Mx X
22
= 1 (() − 2 )
This x 2 equation M Ix resembles 5 MR 2
the standard equation of parabola i.e., (x–h)2 = 4p (y – k) Hence the curve will be parabolic. 28. (c) : The distance to horizon of the tra
d=2
gh d2d 1

= hh
2
⇒ d
X by conse
d= h2h 1 ⇒ h 2 = 4 h 1 29. (d) : The mass number A of the end nucleus is 232 – (6 × 4) = 208 The atomic number Z of AZ
is 90 – (6 × 2 – 4 × 1) = 82
30. (a) : Applying law of conservation of energy 12
21

11

−3 1 20 10 − 3
mv v12 2 A B ( = 20 + 15 mgh × 10 m/s = 12 )( mv 5 ) 2 2 B + ( 20 × 10 = −3 2 )( ( 10 )( × 10 ) )( v2 B ) So, the angular
20
particle about point O is mv B r= 1000
15 10 10
× ×( + ) = 6 kg-m2/s

Potrebbero piacerti anche